Sunteți pe pagina 1din 79

Published by

AMT PUB LISHI N G


Australian Mathematics Trust
University of Canberra ACT 2601
AUSTRALlA
Copyright ,c2001 AMT Publishing
Telephone: +61 2 6201 5137
AMTT Limited ACN 083 950 341
National Library of Australia Card Number and ISSN
Australian Mathematics Trust Enrichment Series ISSN 1326-0170
101 Problems in Algebra ISBN 1 876420 12 X

11 AUSTRALIAN MATHEMATICS TRUST


I!+1!`+! 5+?1+
[D1uRA LuNN11II
Chairman
Editor
GRAHAM H POLLARD, Canberra AUSTRALIA
PEER J TAYLORg Canberra AUSTRALIA
WARREN J ATKINS, Canberra AUST RILlA
ED J BARBEIU, Toronto CINADA
GEORGE BE RZSENYI, Terra Haute USA
RON DU NKLEY, Waterloo CANADA
WALTER E MIENTKA, Lincoln USA
NIKOLAY KONsrANTINoV, Moscow RUSSIA
ANDY LaJ, Edmonton CANADA
JORDAN B TABOV, Sofia BULGARIA
JOHN WEBB, Cape Town SOUTH AFRICI
The book' in this series are selected for their motivating, interesting
and stimulating sets of quality problems, with a lucid expositor style
in their solutions. Typically, the problems have occurred in either
national or international contests at the secondar school level.
They are intended to be suffciently detailed at an elementar level
for the mathematically inclined or interested to understand but, at
the same time, be interesting and sometimes challenging to the
undergraduate and the more advanced mathematician. lt is believed
that these mathematics competition problems are a positive
infuence on the learning and enrichment of mathematics.

!LIALL
Thls bcckecntalnscnc hundrcdhlghlyratcdprcblcnsuscd ln thc traln
|ngandtcstlngclthcL$AIntcrnatlcnalVathcnatleal0lynplad lV0)
tcan |t |s nct aecllcetlcn c!cnc hundrcd vcry dleult lnpcnctrablc
qucstlcns |nstcad. thc bcck gradually bullds studcnts' algcbrale skllls
andtcehnlqucs. Thlswcrkalnstcbrcadcnstudcnts`vlcwclnathcnat-
lesand bcttcrprcparcthcnlcr pcsslblcpartlelpatlcnlnvarlcus nathc-
natlealecnpctltlcns ltprcvldcsln-dcpthcnr|ehncnt lnlnpcrtantarcas
c!a|gcbraby rccrganlzlngand cnhanelngstudcnts`prcb|cn-sclvlngtae-
tles and stratcglcs Thc bcck lurthcr stlnulatcs studcnts' lntcrcst lcr
luturc study clnathcnatIes
W
JN!ULJN
ln thc Lnltcd $tatcs olAncrlea, thc sclcetlcn prcecss .cadln tc par-
tlel patlcn ln thc lntcrnatlcnal Vathcnatleal 0lynplad lV0) ecnslsts
cl ascrlcs clnatlcnal ecntcsts eallcd thc Ancrlean Vathenatles Ccn-
tcst I0 ,AVC I0) , thc Ancrlean \lathcnatles Ccntcst I2 AVC l 2) ,
thc Ancrlean lnvltatlcnal Vathcnatles LxanlnatlcnAlVL) , and thc
Lnltcd $tatcs cl Ancrlea Vathcnatleal 0lynplad L$AV0) Partlel-
patlcn ln thc AIVL and thc L$AV0 ls by lnvltatlcn cnly, bascd cn
pcr!crnanec ln thc prcecdlncxansc!thcscqucnec Thc Vathcnatl-
eal 0lynplad $unncr Prcran V0$P) ls a !cur-wcck. lntcnsc traln-
ln cl 21-30 vcry prcnlsln studcnts whchavc rlscn tc thc tcp c!thc
Ancrlean Vathcnatles Ccnpctltlcns Thcslxstudcntsrcprcscntlnthc
Lnltcd$tatcsclAncrlea ln thc lV0 arc sclcetcd cn thc baslsclthclr
L$AV0 seorcs and lurthcr lV0- typc tcstln that takcs plaee durln
V0$P Thrcuhcut V0$P, lull days cl elasscs and cxtcnslvc prcblcn
scts lvc studcnts thcrcuh prcparatlcn ln scvcral lnpcrtant arcas cl
nathcnatles Thcsctcpleslneludcecnblnatcrlalaruncntsand ldcntl-
tlcs, cncratln lunetlcns.raph thccry. ceurslvcrclatlcns. tclcsecpln
suns and prcduets, prcbablllty, nunbcr thcory, pclyncnlals, thccry c!
cquatlcnsecnplcxnunbcrslnconctry,alcrlthnleprcc!s, ecnblnatc-
laland advanecd ccnctry, lunetlcnal cquatlcnsand elassleal lncquall-
tlcs
0lynplad-stylccxansecnslstc!scvcralehallcnlncssayprcblcns Ccr-
rcet sclutlcns cltcn rcqulrc dccp analysls and earclul aruncnt 0lyn-
plad qucstlcns ean sccn lnpcnctrablc tc thc ncvlec, yct ncst ean bc
sclvcdwlthclcncntaryh|hsehcolnathcnatlestcehnlqucs,elcvcrlyap-
pllcd
Ecrcls soncadvleclcr studcntswhc attcnpt thcprcblcns thatlcllcw
Takcycurtlnc' Vcry lcw eontcstantseansclvcall thclvcnprcb-
lcns
T:y tc nakc ecnncetlcns bctwccn prcblcns A vcry lnpcrtant
thcneclthlswcrkls alllnpcrtanttcehnlqucsand ldcas !caturcd
ln thcbcckappcar ncrcthancnec'
0lynplad problcns dcn' t eraek lnncdlatcly Bc patlcnt T:y
dlcrcntapprcaehcs Lxpcrlncntwlthslnplceascs ln scnceascs,
wcrklnbaekward lrcn thcdcslrcd rcsult ls hclp!ul
Lvcn llycueansclvc prcblcn, dcrcad thcsclutlons Thcy nay
ecntaln scnc ldcas that dld nct ceeur ln ycur sclutlcns, and thcy

v|||
|otroduct|oo
naydlseussstratcleandtaetlealapproaehcsthateanbcuscdclsc-
whcrc Thc !ornalsolutlonsarcalso nodcls ofelcant prcscnta-
tlonthatyou shouldcnulatc, bu thcyo!tcnobseurcthctorturous proecsso! lnvcstlatlon. !alsc starts lnspiratlon and attcntlon to dctall that lcd to thcn Whcn you rcad thcsolutlons try to rc- eonstruet thc thlnk:n that wcnt lnto thcn Ask yourscl!. What wcrc thc kcyidcas Eow ean l apply thcscldeas !urthcr
Co baek to thcollnalpoblcn latcr, and scc l!Y\! eansolvc l t lna dlcrcnt way |Ianyo!thcproblcns havc n|tlplcsolutlons, butnotallarcout| lncd hcrc
Al|tcrnslnoldaeearcdcncd|nthc Glossary. Lscthclossary and thcrcadustto !urthcryour nathcnatlealcdueatlon
lcanln!ulproblcnsolvlntakcspraetlec lon'tctdlseouracd i!youhavctroublcatrst loraddltlonalpraetiec. usc thc books on thcrcadlnllst

ALKNWLLLNLN5
|hanks to Tlankal Llu who hclped ln proo!cadlnand prcparlnsolu-
tlons
Vanyproblcnsarcclthcrlnsplrcdbyorcdonnathcnatlealeontcsts
|ndlcrcnteountrlcs and !ron thc!ollowujournals.
High-School Mathematics, China
.
Revista M atematica Timioara, Romama
Kvant, Russia
W d dourbcstto eltcallthcorllnalsourecso!thcproblcnslnthcsolu- c \
l tlonpart Wccxprcss our dccpcstapprcelatlon to thc orua proposcrs
o!thcproblcns.
'
A!LVJAJN5 AN NAJN5
Abbreviat!ons
AESVL
AlVL
AVC0
AmCl 2
ARVL
lV0
LSAV0
V0SP
Putnan
St Pctcrsbur
Anc|eanEih Sehccl \athcnat|es
Lxan|naticn
Anc|eanlnv| taticnalVathcnat|es
Lxan|nat|cn
Ancriean \lathcnat|cs Contcst
Ancriean\!athcnaties Ccntcsi l2,
whieh .cplaecsAIlSME
Anc|ean Rc|onaI Vathcnat:es Lcauc
lntcrnat|cna| \Iathcnat|ea| 0lynpiad
Ln|tcd StatcscAnc|eaVathcnat|eal 0lynp|ad
\lathcnat| eal 0|ynp|adSunncrPrcran
ThcWill|an |cwcll Putnan `Iathcnat|eal
Ccnpct|t|on
St Pctcsbu ,Lcn|nad) \IathcnatieaI
0|ynpiad
Notations or Numerical Seis and Ficlds
thcsct c||ntccs
thcsct c|intccrs nodu|c n
thcsctclpcs|tivc|ntccrs
thcsctcl ncnncat|vc |ntccs
thcsctclat|cnal nunbcs
thesctc pcs|t|vcat|cnal nunbcs
thesctclncnncai|vcatIcnal nunbcs
thc sctc| -tuplcs cl .at|cnalnunbcrs
thcscto!cal nunbcrs
thcscto|pcs|t|vcca| nunbcrs
thcsctc|ncnncat|vcrca| nunbcs
thcsctc| -tup|cs c|ca|nunbcrs
thcscto|ecnplcxnunbcrs

g
7

COfJtfJ5
PREFACE
INTRODUCTION
ACKNOWLEDGEMENTS
ABBREVIATIONS AND NOTATIONS
1. INTRODUCTORY PROBLEMS
2. ADVANCED PROB LEMS
vii
ix
xi
xiii
11
3. SOLUTIONS TO INTRODUCTORY PROBLEMS 27
4. SOLUTIONS TO ADVANCED PROBLEMS 65
GLOSSARY 131
FURTHER READING 137
P
f
<
'

!TU\` !1\tN''

|. JN!UL!Y !LN5
Problem 1
Let U. | and C be real and positive parameters. Solve the equation
Problem 2
Find the general term of the sequencE ciefilwc! b' Q * 3. J * 1 and
for aiI'll E N.
Problem 3
Let Xl' X2 - J be a sequence of integers such that
(i) -!_ :_ 2 for i " 1 2 H.
(ii) Xl + .r2
. + ._ - 19:
(iii) +.. t .; =99.
Determine the minimum alld maxil1lum possible values of
-
Problem 4
The function f. defild by
1
.L + /)
,,( * "
C.I + d
where U. b. l anc dare nOllzero real numbers. has the properties
J(19) -
|9 1(97) - 97. and 1U(J')) * *
for all values of .. except -
C
Find the l'allge of f.
"
4
........._
.

_
t
_
o
_

___
t
_
o
_o
____

ProbIem
Prcvc :hat
(a - b)2 <
a +b

_< ' c- b)2


8a

. -
8b
lcr alla _ b >
Problem 0
$cvcral at lcasttwc) ncnzcrcnunbcrsarcwr|ttcncnabcard 0nc nay
crascany twc nunbcrs say a and b, and thcn wr| tcthcnunbcrsa +
andb - .nstcad
Prcvc that thcsctcl nunbcrs cn thc bcard. altcr any nunbcr cl thc
prcecd|ncpcrat|cnseannctec|ne|dc w|th thcin|t|alsct
Problem ?
Thcpclyncn|al
nay bcwr|ttcn |n thclcrn
whcrcy x -I and a,sarc ecnstants
l|nda
2
'
Problem8
Lcta, b, and L bc dist|nct ncnzercrcal nunbcrssueh that
Prcvcthat label ~ I.
Prob\em0
1 l 1
a + - b +- ^ C +-
b e a
F|ndpclyncn|alsf(x), g(x), and h(
x
)
, |lthcy cx|st. sueh thatlcr allx,
{ I |lx
<
-1
If(
x)l- Ig(
x
)/
+ h(
x
) = x -. |l-I _ x _
-2x +. |lx >
Problem J0

P|ndallrcal nunbcrsx lcr wh|eh


3
-
+2
-
I 2 +I 3-

=
1
Problem J
I
Pindthclcastpcs|t|vc| ntccrJ sueh that

<
J
|crallpcs|t|vc|ntccrs.
Problem I2
L t b L d ande hcpcs|t|vc |ntccrssuehthat
c a, , , ,
abcde a -b +e-d +C.
lindthc nax|nun pcss|blcvalucclnax{a, b, C d, e},
Prob\em I3
}valuatc
!
1
.l

t.+.!
+
3
+
.!+3-1!
-
'
+
19991 +.!+.l.
Problem I4
Lctx ~ -a-l
-
Ja2 - a +t. E l
lindallpcss|hlcvalucsclx,
Problem I
l| ndal lrcalnunbcrsx |cr wh|eh

0
-
+

+ I 2
-
I 3
-
+ l1
'
r
i

_4 ~~~@~@~@~@~~~

~

'+

'o

y+
o

'

' o r
yr
o1

....
..~~~~~~ 5
ProbIem I0
Let
!
N ? N N be lunetlon suehthat
!
, l l) !.
!
,m+l.o) !,m. o)+ mand
!
m. -l ) =!
,m. o)-
lor a|lm.oE No
llnd al| palrs(j ,) suehthat!,j ,)
!cl
ProbIem I?
Let] bealunetlonde|nedon ,. l suehthat
!,)=!,\) tand 'a) !'|)|< a ||
loralla= |ln the lnterval ,l.
Prove that
ProbIem I8
l
!, a) !,|) | < _
llnd aIpalrsollnteers :,,) sueh that
:

- ,

= ,: - ,)

ProbIem I0
!
Let
!, ) ~lor real numbers :
^
-
-!

Lva|uate
ProbIem 20
Prove that lor o_ 1 the equatlon
has lnteersolutlons
ProbIem 2I
l l l
-

- -= l
: : :
llndallpalrs ollnteers ,a |)suehthatthepo|vnomlala:

-|
:
-l
ls dlvlslb|eby :
- :- l

Pro11r 22
Q`
poslt|ve lnteero. l etj,o)betheproduetolthenon-zerodllts
o`
n
l! hason|yonedllt, thenj, o) |sequal tothatdll t ) Let
5
j, l)+ j,!)-

-j,)
What lsthelarest prlme laetor ol5`
Pro11+r 23
Iet:,beasequeneeononzeroreal numberssueh that
:-
-

:-
:--
!:-z Xn-l
lor ! J^
}stabllsh neeessary andsudelent eondltlonson : and :

lor : to be
anintegerlor |nhnltelymanyvaluesolo
Pro11r 24
$olvetheequatlon
Pro11r 2
PoranysequeneeolrealnumbersA [a,.a

.a

,dehne Ato be
thesequenee[a

-a,.a

-a

.a

, . . $upposethatalloltheterns
othesequence A; are l, andthata

.=a.

P:nd_-
Pro11r 20
PInda|lrea| numbers:satlslylntheequatlon
Pro11r 2?
Provethat
Pro11r 28
!

-J
-
|
-
-
g-

- 1.
1
l1 < _ I
< 17
.
k=l

Deternlnethenumberolorderedpalrsollnteers,m o)lorwhlehmo _
0 and
m

-o

-
mo=JJ
,


7
gggggggg ____
'
_
+o

__
'
_
o
_
+
_
o
_ _

ProhIem 20
Lct a,|. and c bc posltlve rcal numocrs sueh that a+ | -L 4 and
a| + |cca 4.
Provc that at |cast twoolthc lncqua|ltlcs
arctruc
Problem 30
Lvaluatc
ProbIem 3I

a | 2,
|- e 2,
c- a 2
n
1
`
,r- /)!,+ /)!
k=O
Lct0 < a< 1. $o|vc
lor pos|tlvc nunbcrsx.
ProhIem 32
Whatls thceocdelcnto|~
.
whcn
(1 -x -2x)(1 + 4x) \-2nx)
lscxpandcd?
Prob!em 33
Lct 6 and rbcdlstlnet posltlvclntcgcrs
llnd thc naxinun vauc ol Ixm - xnl, whcrcx lsarcal nunbcr|n thc
|ntcrval (0,1).
ProbIem 34
Provc that thcpolynonla|
whcrca,.a
.
.' '
.
,a-arcdlstlnetlntcgcrs,eannotbcwrlttcnastLcprod-
uet oltwonon-eonstant po|ynonla|s wlth lntcgcr eocde|cnts l c , lt ls
lrrcduelblc
Prohlem 3
| h

h
. 11 d rcdpalrs o|rcalnunbcrs(x. y or \ .
Fmd a
or c
, -x)(l -x
.
(+ x4) \-y7
and , - , I -
.
, -
,

7 -x7.
Prohlem 30
$olvcthccquatlon
.
) 2
x+1
2
2(2X -

): -(2 - 2 x 7
lor rcal nunbcrs x.
Prohlem 3?
t th n \
Lct a bc an irratlonal nunbcr andl ct7 bcanlntcgcr grcacr a
Provcthat

a-
,
- a- a
.
- 1
lsanlrratlonalnunbcr
Prohlem 38
$olvc thc systcn olcquatlons
Prohlem 30
,~,
~
~
.
+ ~

.
,~. - ~
+ ~,)
.
,~
- ~,-~,
.
,~,- ~+ ~\)
,~, - ~,-~
.)
~.
~,-~, ~
.
~,,~,-~, ~

7 ~,,~,-~.- ~,
= ~z,~
.
-~. ~s
~,, ~
+ ~,- ~;
Lct , andz bc eonplcxnunbcrs sueh that
and
r _
N1tLLH5.

AA'-.L"1^.
HI1!\! K :
I
.. ,
~,
= .~'
~+ Y z 7 2,
x
.
-
.
-z
.
= 3
~z = 4.
\ I
1
- -
z - 1 y
z -x-I zx + -1


I
B \. |t'+ot!o+j +o11s
ProbIem 40
Vrlatlsgolngtopiekthrccnon-zcrorca|nunbcrsand\Ir Tal|sgoln i
toarrangcthc thrccnunbcrsas thceocdelcntsolaquadratlecquatlon
o.

Vr lat wlns thc ganc ll and only ll thc rcsultlng cquatlon has twc
dlstlnetratlona| soIutlons
Whohasawlnnlngstratcgy?
ProbIem 4I
Clvcn that thc rcal nunbcrs a,b, C ), and c satlsly slnuItancousIy thc
rclatlons
a|C)c = 8 and ab
2
)c={b,
dctcrnlnc thc naxlnunandthc nlnlnunvalucola
Problem42
llndthcrcalzcrosolthcpolynonlal
whcrc alsag|vcnrca|nunbcr
ProbIem 43
Provc that
I 3 ,- I I
- . -... .<
, 1 ,!
v
loral| pos| tivc lntcgcrs!.
Problem 44
Lct
P(x)=a,+

a:
-
a

bc a nonzcro polynonlal wltn lntcgcr eocdelcnts sueh that P,r)


P,o) 0 lor sonclntcgcrsrando. wlth 0
<
r
<
o.
Provcthata,_ " lorsonck.
ProbIem4
Lcttbcaglvcnrca|nunbcr
llnd a|leonplcx nunbcrs sueh that
\J, \J,
=tt.
L
9
t

!o


mm

1. |n!r
Pro11
r40
d
Thcscqucnecglvcnby, a ::
|.an
lspcrlodl e
., = - -
,+
;

,
-
Provcthata|=I
Pro11r 4?
d )bcrcalnunbcrssuehthat
Lct ab, c
an
a
b
2

l ) ()- I) > ae|)- I)


Provc that
Pro11r 48
Plndalleonplcxnunbcrs : sueh that
'`:I)(1:I)(b:

I)I`:I) =,
Pro11r 40
dlncrcnt lron I ol thc polynonlal
Let ,

, ,-
,, bc thc zcros
P)~

~ I, ! _ ,
Provcthat
Pro11r 0
I I
n-J

I-,=,
=
`
I- , I

Lctaand|bc glvcn rca|nunbcrs $olvc thcsystcnolcquatlons


X - y
I- + y
2
y - x
_I- x
2
+ y
2
lorrcal nunbcrs andy,
a
= b
/
_\/T\1 !1\1N'
,H ,:
_ ,.+ !H
)
b
l
J. AVANLL |!IlLN5
Prohlem51
Evaluate
.
. . .
.
-
+ 8
-
+ .
Prohlem52
Let r,,.bcposltlvcrcal nunbcrs sueh that+ , +
= l

Determine with proolthc n|nlnunva|ucol


Prohlem 53
Find all rcal solutlonsto thccquatlon
.

-
,
+
,
:

Prohlem 54
Let [:

),

bcascqucnec sueh that:, .anc


for all n E N.
Find an expliLit lornua|or:
-

Prohlem 55
Let r, ,. and bcposltlvcrca| nunbcrs Provc that

:
--

,: ,)(: - ) , - ,, - )(, - :)

- : l
+ , + :) ,
,)

\.....ggggggg t t t+ o11
Prot1r 0
lind. wlt| rool, al| nonzcropolyoonialsJ(z) sueh tat
,:
:
)+ J(<)J( + 1
) o
.
Prot1r 57
LctJ . N N bca|unetion sueht|atJ(n + 1) > J(n) and
orall7
Laluatc J(2001).
Prot1r 58
|],n)) = 3n
Lct F bc tnc sct o|all olynomials J(1.') withiotcgcrs cocme|cnts su

|
t|at J(x) l hasat lcast ooc iotcgcrroot
lor caeh .ntcgcr k > 1. nd Tnk, thc cas: |ntcgcr grcatcr ihao 1 |or
whle| thcrc cxists
f
E F such that thecquat:on f(x) |ascxaeiy
k d|stinet | ntcgcr oots
Prot1r 59
LciJ_ = 2 and
|orn _ !.
Procthat
r
--
, = x; -J 1,
1 1 1 1 1
1- *<-
+
-
+
"'
+
-<1
-
_
2
2
"-1 J] J_ jy 2
2
" .
Prot1r 60
$upposc t|at J I+ 2
-
is a deercas|og |voetioo suc| :|ai |or a!
J y E I+,
J(.1: + y)
+ J(f(x
) + J(y)) J(J
j
+ fry)) + fly +
J
(.z:))).
Provc :nat J(f(x)) = x.
l
2. Attt +o11
Pro11r 61
.
11 l
tlons f ] ~ ] sue|thai
Fmd a
une
lora|lx
. Y E
Pro11r62
Lct <a < 1.
f(x
+ y) + f(x
-y) = 2f(1') + 2f(y)
Provctnatthccquat|on
x3(x + 1) (x + a) ' 2;r -:)
\5
haslourdlstlnetrca|solut|onsand|odhcscso|ut.oosi ocxol|e| |orm
Pro11r63
Lcta, |,andbcposlt|crca| numbcrs sue| that :-= 1.
Provc that
Prot1r 64
Find alllunetlons f, dcncd oo thc set o| ordcred pa|rsolposit|vcintc-
gcrs satls|ylngthc lo|low|ngpropcrtles.
f(x, x) = x, J(x, y) = f(y, ' ; (x + y)f(.z:. y) = yf(:r. x + ,`
Prot1r 65
Considcr n eomplcx numbcrs Zk. sue| :hat I = : 1. k 1, 2" " . n.
Provc that thcrc exisi Cl. C2, .... Cn E [:l } sueh tha. |or any Tn : n,
ICIZI + C2Z2 + . ` . +Iyyl : 2.
Pro11r 66
Plndatrip|colrat.ona|numbcrs ,a. |, suehthat
\6 2. Attt +o11

FrobIem 0?
l|ndthcnln|nunol
whcrc ,.

.
-
arc rca| nunbcrs | the intcrva| ,. l )
FrobIem 08
lctcrn|nc -

+ ,

+ | l
-

+
_
+
_. , _
@ ;

Froblem 00
linda|| lunet|onsf P P suehthat
,-,-
`

f(y))
= ,,: ,

+ y
lor a|| - , E R
FrobIem 70
Thcnunbcrs 000. | ,
_
999 havcbecn wr| ttcn on aboaru
Laeht|nc, onclsa||owcdtocrasctwonunbcrs, say, :and|, andrcp|aec
thcnby thcnunbcr

nln a
A!tcr 999 sueh opcrat|ons, onc obtains cxaet|y onc nunbcr e on thc
board Provc :hat c< l
Froblem71
Lci:,. :

, .:
-
bcrcal nunbcrs, nota zcro
Provc thatthccquat|on
hasatnostoncnonzcrorca| root
c
_Att
t +o11
Prohlem
72
| ] bc thcscqucnecolrca!unbcrsdcllncdby:

and
Lct :

:
-

,( :
-
:
lor _|
Por how nany dlst|net a|ucso!dowc havc :.
,,

0
Probl m 73
a) lothcrccx|st!unetions

P* P and g . P -P sueu tnat


f(g(:r))

and g(f(.r))
* -

lora|| -E P
b) lo thcrccx|st 'unetlons. P P ad g 1 P suehthat

.g. -)) = -

and g(f(x))

lor a|| -E P`
FrobIem 74
\7
Lct0< :

_:

:
-
.0 < |

_ -

_-
-
bccal unbcrssuehthat
- -
`
:


_
.
)=} z=1
5upposcthat thcrccx|sts _k _ suehtnat| _:.!or _i _k and
',:,lor i > k.
Provc that
Problem ?
0:vcn c,ht non-zcro rca| nunbcrs:,.:

:,provc tnatat |castonc


olthc lo||ow|n, s|x nunbcrs :,::

. :

:, :

:_. :

:,:,+ :

:_. :,::,:

.:,:-+ :_:

lson-nc,atlvc

Problem ?0
Lct: | andcbcposlt|vc rca| nunbcrs suehthat :-*
Provc that
L
< |
:
:

| +c

,+ c :+ ca

b
B 2. Attt +o11

ProbIem 77
llndall |unetlons1
` I I sueh that thccquall ty
1
,
1,
;
`+ y) =1
r
.
`|
1,r
`
holds |orallpalrsolrcal nunbcrs r,:
ProbIem 78
$o|vc thc systcn o| cquatlons
ProbIem 79
J
' )
r = J
r
. +

.
:+J

~
-0
r
. +
J
.
I. lat and\Ir Ta| play agancwlthapo|ynonlalo| dcgrccatlcast|.
;
.
+ _
r
.
*
_~
..
+
_r +I
Thc

llln rcal nunbcrs tocnpty

spaecsln turn || thcrcsultlngpoly-


nonial hasnorcalroot, \Ir Fat wus; othcrwlsc, \IrTa|wlns
l| Vr latgocsrst. whohas awlnnlngstratcgy?
ProbIem 80
llndi! posltlvc lntcgcrsk |or whleh thc |ollowlng statcncnt l struc: i|
F(r) rs apolynonlal w!th lntcgcreocmelcnts satls|ylng thc eondltlon
O$F(c)$k |or c=O.L

. 1+|,
thcnF(O) = !,))
F(k + I )
ProbIem 81
T|cl:bonaeelscqucnecF- l sglvcn by
Provcthat
|oralln _ 2.
L
@Attt +o11
9
Prohlm 82
P|ndall |unetlonsu: I I |or whle| thcrccxlstsastrletly nonotcnle
u
nctlon1
: I -- I sueh thai
1

-+ =
1
r
`
u

1
,
|orallr ,)
E R
Prohlem 83
Lct:, .:
. , :
bceonlcx nunbcrssuehthat
Provc thatthcrccxlstsasubsct 5 o| :,._
suehthat

zES
Problem 84
A polynonlalI,r}o|dcgrccn
,
wlthlntcgcreccmeicntsand7 di.i lnet
lntegcrrootslsgivcn
Plndalllntcgcrrootso| I, I(r givcnthat+isarcoto|I,.
Problem 85
Tworca|scqcnecs_ J_ . . .and)
. ).

. ,arcdcnncdlnthc|ollcwin
way
and
)
)+, =
.
I+ + )
|oralln _ Prcvc that2 < :

< 3ffrall > l.


ProbIem 86
F
]
orapolynon|alI(r),dchncthcdiference of I(:)cnthelntcrval,

. ,O /- I`
P

vcthatitlspossiblcto disscet thc |ntcrval + ., .nto ahnitcounbcr


atcrvals and eolorthcnrcdandblucaltcrnatclvsuent|at |c: cvcrv
quac
.
.

rat,e polynon:al I(.i ) thc tctal dincrcneco| I(:) cn rcdintcrvals


IS cqualto that o|I,. on bluc lvtcrvals
Whatabouteublepolynonials
20
FrobIem 87
Civcnaeublecquatlon
d
X + - : 0,
2. At:tt P+o11
^
\lr lat and Vr Tal arc p|aylng thc lo||owlng gac ln onc novc. \!r
lat ehooscsarcalnunbcrand Vr Talputs .i lnonco|thccnptyspaecs
Al:cr

thrcc noves the ganc is ovcr \Ir lat wlns thc ganc il thc lina|
cquat|on has thrccdlstinetlntcgcr roots
Whohasaw.nnlngstratcgy?
FrobIem88
lct > 2 bcan lntcgcr and l ctf z ] bcalunetloo sueh that |cr
any rcgularn-gon A,.:.A,
Provcthat f ls thc zcro |unetion
FrobIem 89
ctp bcaprincnunbcr and |ctf-) bc apo|ynonialo|dcgrcc d with
utcgcr eocmelcnts sueh that
l)
1(
0) 0, f( l)
. l) |or cvcry poslt. vc .ntcgcr 7 thc rcnalndcr uon dlvls.on o| f(1/.)
by p lsclthcr0 or
Provcthatd : p - 1.
FrobIem 90
Lct bcag.vcn poslt.vc lntcgcr
Consldcr thcscqucnec:,:

.:
-
with:, and
2
lor / 1, 2,
Provcthat

i -:
-
i
n
2. Atnt +o11
2
Prohlem 91
: bcnonncgat.vcrca| nunbcrs oota| |zcro
]et:
`

-
)
Provc tha|
,- :

-
-

.
= :
-
,:~ :, 0 has prceisc|y onc
posltlvcrca|rooiP
b) Ict A _

:,and B __
.
:,
Provc that A
~
u
Prohlem92
Provc that thcrc cxlsts polynonlal !, -.y) wi:h cal eocmeicnts sueh
that ,-y) 0 lor alrcalnunbcrsJ and y, whieheannotbcwr. ttcn
H the sunolsquarcsolpoynonia|swithrcaeocmeicni s
Prohlem 93
lor caeh posltlve lntcgcr

show that thcrccxlstsapositlvc lntcgcr /


such that
/ f(l:) + ,
-
+ ;,`
-
+ 1)
lorsoncpolynona|s f, ;wlth .n:cgcr eocme.cnts, andhnd:hcsna|lcst
sueh /asa|unetlon olH.
Prohlem 94
Let x bcaposltlvcrcal nunbcr
) Provc that
b) Provc that
7! - .) |
:+ 1) j +n)

r- .) |
(-+ : ..
.
. (x
+ )

(-+ /
`

ce dr ob| e

FrobIem 95
lc: n 2 bcan lntcgcr.andlei
beasctol
,
clcncnts
Prcvcthatonceanhndnlnc dlstinet nunbcrs, .,. C i " 1. 2. lnX
-ue| :hat thc systcn
,.+ ,,+ ,.
,+ ,+ ,.
,.+ .,,+ ,.
o
o
o
has asolu:lon-,

,.,in nonzeroin:cgcrs
FrobIem 96
Lci H 2 bcanintcgcrandlct-,, ,beposltivc rcal nunhers
,
1
$uppcsc that _~

_
,
|-

Provc thai
,
+
, +
2


v v
FrobIem 97
Lc:-,. -,.

.,bc dls:lnet rcal nunbcrs lchvc thcpcyncnlals


/ -- .- , ,|' -,
and
/-

-
-,

-
-,
,
-

+
X ,
Lci
., ,.,bc thc rootsol
$howthat
@Advaced rob|ems
Prohlem 98
.
h t loranv posltlvc .nicgcr n. thc pclynorv:al
5hovt a

.
,
.
!'`
+
x) +

3
b
rlttcn as theproduetcltwc ncn-ecnstan: o|ynonials wlih
cannot cw
|ntccreocdelcnts
Prohlem 99
!ct]]] R-Rbc!uneticns-ueh:hai
,
:.,),+ ,),
ls neaotonle|orall,.,.,E l
Prove that:hcrccxls: ,,,E l uci all zcro. sue| that
foral|-E l
ProbIem 100
Let -,.-,.-,bcvarlablcs and !ct

,.,. .
,.,bc thc suns o!
nonenpty subscts o! J_
!ct,,-,. -,bcthckth eIementarysymmerrle olynom!aIln
theYi ,thc sun olcvcry produe: o|k dlst.net,s).
orwhlehk and H iscvcryeocdelcnto|,,, as apolyncnla|ln-. .-,
cvcn:
lorexanplc. l
_. t|cn ,

,. ,arc [ , .
and
ProbIem 101
,,= ,
+
,+ , 2J:l + 2-,.
-
,
,-,,
+
,, J'i + +
.,
-
,=
..,~
XX2 + :,.
!rovc that thcrc cxl.t 10 d|.tlnet real nunbcrs [.,.

,,suen that
the
equatlon

-
,-- , , J ,,= (.: + ,-+ ,
-,,`
hascxaetly 5 dlllcrcntrcalroots
'\U1T'
!TU\` !1\tN'
! ! 1 { `-;'
.. m - | = , . ___ ~
J

J 0 c 0 0
.
. 5LUJN5
JN!UL!Y !LLN5
Prohlem
1 Boman!a I0?4|
leta, |, and e bc rcaland posltlvc paranctcrs
SoIve thccquatlon
+|:+v! + e:+ve

: v!:+ ve !:+ , c:

5olution 1
Its easy to scc thatx isa solutlon $|nec thc rlght hand slde lsa
decreasing !unetlono!:and thc lc!t hands|dc ls an |nercas|ng |unet|on
o|x, thcrc lsat nostonc solutlon
Thus x = 0 lsthconly solutlontothccquatlon
Prohlem 2
nd thc gcncral tcrn o! thc scqucnecdcncdby r, 3. r, = A and
|oralln E N.
5olution 2
Weshall provc by lnduet|on that r- = n +3
Thc elaln |scvldcnt lor
n = 0, t
or / 1, llr,, = /+ 2 andr,7 /+3, thcn
r,
+, 7 :j,
/r,= (
/+2)' /(/3)= k

A
deslcd
1hsconplctcs thc|nduetlon
-
2B
ProbIem 3 AHSME 1999J
3. So1Joto |txotoxy xo11,j
~
LetJ]+ +
`

..+ be asequeneeollntegers sueh that


l) -|:,2!:I2. ., n;
ll) + +

-:|J,
(
) ` +-+
`
-
.

.
-+ J.
leternlne the nlnlnun andnaxlnunposslblevaluesol
:-+-
.
:.
SoIution 3
Let : bandc denote the nunber ol -1s 1s and ?s r n th
t l W

. e sejuenee
'
espee rvey

eneednoteonslderthezeros Then: b c arenonnegat

ategerssat:slylng
. v
. b-2c|J and - b+ 4cJ J.
ltlollowsthat. 40-c andb J-3c where0
<
c < IJ
.
b > 0)
so
.
_ saee ,

-
+

:
`
-

+: -:- b-c IJ +0c.


Whenc 0 (: 40

b J), the lowerbound(IJ) lsaehleved


Whenc |J(: 2 |b 2). theupperbound( |33) lsaehleved
ProbIem 4 AIME 1997]
The |unetlon ]dened by
!'
:)
:: - b
c: -I
where :,b,c and I are nonzeroreal nunbers,has the propertles
]('|J J
(
''J7 and J( |(:)) :.
lor allvaluesol:, exeept
llndtherange ol].
c
SoIution 4, Alternative 1
lor all:](](:)) J l e ,
.. -b_
: -b
c:-d
..+b_
:

c -I
c: + I
].
5o1
'1oto |txotoxy xo11
w
le
le
:- bc):+b(: -I)
= = :.
c:-I): - b -I`

c(:+ I|:`-(I` - :`|: bI| 0


which |pllesthatc(: -I) 0 $lneec ; 0. we nust have : =-I.
T hecondltlons ] ( IJ) IJ and J( J7) J7 leadto theequatlons
and J7`c 2J7u -b

ence
(J7 - |J}c =2(J7 IJ}:.
It|ollowsthat : c whlehlnturnleadstob -I 43c.Therelore
]:)
: -I 43
~ -

1 :
which neverhasthevalue.
Thustherangeol] lsR {.
5olution 4, Alternative 2
29
T he statenent inplles that ] ls lts own inverse The lnverse nay be
|cuadby solvlngtheequatlon
fory. Thlsylelds
:- b
:-
c + I
]
1
(:)
I: -b

-::-:
T he nonzero nunbers:,b, C and I mus: therelore beproportlonalto I,
-', "c and :, respeetlvely; l tlollowsthat : -I, and therestls the
measlntherstsolutlon
Prohlem 5
rovc that
for a ._b > 0
:-b)` :b (:-b)`
< - v :b < -- -
'
: 2
-
b
8olution 5, Alternative 1
N
ctethat
_ -v_

< I
<
_- \_

2
- -
2\
30
3. So1'Jot'o |t'+ot'o+y +o11(,j
| e
l e

!)
:
2

!

!)
:
= ~ < < = ~
8 2

!
|ronwhleh theresult|ollows
SoIution , AIternative 2
Xote that
Thusthedeslredinequalltyisequlvalent to
A_+ !-2. _A!

whleh |sevldent as_!> 0 wh|en lnplles_ ._ |)


FrobIem 6 St Fetersburg 1989J

$evera|(atleast two)nonzeronunbersarewrlttenonaboard 0nenay


eraseany two nunbers. sayand!, and then wrlte thenunbers+

and! -_ rnstead
Prove that the set o| nunbers on the board a|terany nunber o|the
preeedlngoperatlons. eannoteo|ne|dewlth the ln|tlal set
SoIution 6
let5 be the suno|the squares o|:he nunberson the board Xotethat
5 lnereaseslntherst operatlon anddoesnotdeereaseinanysueeess|ve
operatlon,as
wlthequalltyonly l| = ! = O.
Thls eonpletes the proo|
o1 t'
+
o
t

'

1+

@@@@@@@@@
3
],
5o1 ot
.
11+r ? A|ME 1986J
r0
The pclynolal
: 3
16 17
1-:1 + x -x + +x _
my be
wrltten ln the lorn
: 16 + 17
)+al Y + : + + a16Y
17Y ,
h
x + 1 anda,sareeonstanis llnd:

werey-
So1tio: ?, Alternative 1
Let I(x) denotethe glven express|on Then
and
O
3
1
8
xf(x) = -r + r , -x
( 1 + x)f(x) = 1

J
J!
Hence
1 (y " I ) 1 (y " 1)
1
8
f(
x) =f(y-
l
)=
1 + (y-l) y
Therefere
:
lsequal to the eoede|ento|y3 ln the expanslon o|
i.e.,
So1tio: ?, Alternative 2
Let f(x) denote the glvenexpresslon Thcn
T hs
f(x) = fey -l)= l -(y 1) + (y -.)
..
.
-(y ;
.
'
:

'
)
1
7 = I+J y) + ,I -y) +

+
y .

:=
,_,

,,
~
,

,
Here weused the|ornula
n

l
1

k1 k+ l
and the|aet that
'
=
'
=
'

4
2 4. So1'Jot'o |t'+ot'o+y +o11,j
Pvot!+r 8
Lct a. b, andc bc distinet iionzerorcal nunbcrssueh that
Provc that labcl = 1.
Solut!on 8
1 1 1
a + -=b + -=c + -.
b c a
lron thc g|vcneonditionsit follows that
b-c c-a a-b
a - b = -
b
- b - e =
- andc -a =

c ca a
~
\lult|plyingthcabovecqua:|onsgi.cs(abc)2 = 1, !ronwh:eLthe dcsirc
rcsult |ollows
Pvot!+r 9 Put:ar 1999]
l|nd polynonials f(x), ),.,, and h(x). ||ihe exist. such that or al J
1''`
,),r, + hex) = 3.1: -2 if -iS x S 0
{ -1 if.r < -1
-2:+2 if.r>O.
Solutlon9, Alternat!ve 1
$|nec r = -1 and J = 0 arc thc two erit |eal valucs o| :he absolut-
|unet|ons, onc eansupposc that
I,.) = alx +ll+bkl +cx +d
{ (c -a -b).r. + d -a ilx <-1
(a+c-b)J: + a + d if-1S.1:S0
(a + b + c)J: + a +d ifx>O.
wh.eh inplics that a = 3/2, b = -5/2. c = -1. andd = 1/2.
Ecnec f(x) = (3x + 3)/2, )(,= 5.r/2, and h(x) = -+
Solut!on 9, Alternatl ve 2
Xotc that|!rex) and sex) arc any two!unet|ons. then
r+ o -r - o
cax(ro) = 2 .
Therc!orc.i|F(x) |s thc g|vcv |unetion wc havc
F(x) nax{-3+- 3,O} - max{5J:.O} + 3:+ 2
-3-- 3-
|3:+ 3, ) /2 - (5x + 15:1:1)/2 + 3x + 2
|
(3+ 3)/21 - 15.r/21 -J -_.
'
o1t+

'o

4
],
.
)ro11+r
10
.
' d
ll rcalnunbcrsx !orwh:eh
!m

8x +
2
7x 7
12x + l8x 6
So1tio:10
tt` g2
x a and 3
x =b. thccguatiovbceoncs
Byse 1
.
i.e.
ie.
ie.
a
3
+ b
3 7
a2b + b2a
-
6'

6a2 -13a+ 6b2 =c.


(2a - 3b)(3a - 2b) = O.
Th l
2
x
+ \
3
x+1
or2
x-1 -
3
x-1 whieh implies thatr = -1 and
crcore = -
,
x = 1.
It|scasytoehcekthatboth.r = -l andx = 1 sais|y thegivcncquation
Problem II Boman!a 1990]
llnc thc |castpositivc intcgcr sueh:ha:
for all positlvcintcgcrsn.
5olutlon 11
N0te that

<

<

`
+
,

+
+
,
= ( 1 + 1 ,
=

and |orn = 5,
1hus m =
44 4. So1Joto |trotory ro11(_

Froblem 12
Lct :. b, C, I. andc bc posltlvelntcgcrssueh that
:bcot :- b-C o -c.
l|ndthc naxlnun posslblcvaluco|nax{:.bC.oc.
Solution I2, Alternative 1
"
$upposc that : _b _ C _ o _c. Wc nccc to nd thc caxlnun valuc
0|
t. $lnec
c <:-bC -o+c _ c,
thcnt <:bcot _c, l c 1 <:bco_.
|cnec(:bC, I|,1I1 =| (1 I 3|(1.1I4) (1 12 =| or
(I.11.), whleh lcads tonax{t

Solution 12, Alternative 2


Asbclorc, supposc that : _ b _ C _o_c Xotc thai
1 I I 1 1
1-----
bcoc coc : ot:b c :bc :bco
1 1 I I I 3o-c
<---------
-
ot oc oc c o oc
Thcrclorc,oc_ 3-I c or(o-l}(t-I| _4
l|o 1 thcn : b C I and4+ccwhleh lslnposslblc
Thuso I I anc t -1 _ 4 ort _ .
ltlscasy toscc that(1 1 I 2| lsasolut|on
Thcrclorcnax{t
Comment: Thc sceond solutlon ean bc uscd :o dctcrnlnc thc nax|
nun valuc o| {r,.r

.. .1 whcn z, .r

.. J arc posltlvc lntcgcrs


sueh that
Froblem 13
Lvaluatc
3 4 2001
l! -2!-3!
-
2! 3'-4!
-

-
1 J J J!-2000!7 200I!
.
o |trotory ro11(
].
5o1'+ot

5olution 13
Note that
I-2
I2
I! -(I + I|!-(I-2|! I!l -I-I(I l}(I-2|
,
I
I!(I- 2|
I t I
(I-2|!
(I- 2) -I
(I+ 2|!
I
(I-I|!
I
(I2|!

Byteleseoplngsun. thc dcslrcdvaiuc lscqual to


2
-
200l!

Prohlem 14
Let x=

- - l ~

:-: + I, aE R
Find all posslblc valucso!x.
5olution 14, Alternative 1
Since
,a- ] :!
-1 > ] :!
and
2:
x-
,
-

a-: -I -

a -:-I
we havc
Ixi <,2:/:]2.
5uarlagbothsldcsol
-, a - a -I , a : 1
yields
4b
36 3. So1'Jot'o |t'rot'ory ro11(,j
"
$quarlngboth sldcs olthc abovc cquat|on glvcs
$lnec:`_ . wcnusthavc
$lnec I xl <2,x`

< whlehlorecs:` i <

Thcrclorc,-I <x < i.


Convcrscly. lor cvcryr (-I,i ' thcrccxlstsarcal nunbcra sueh that
r= , :`-c I -, :`-: + i
Solution I4, Alternative 2
Lct A = (-l,2,(,2), P = (I/2,(/2|. and 1 = (a.

) Thcn P
|s a polnt on thc r-axls anc wc arc looklng |or all posslblc valucs ol
c= 1A1P.
By thc Tiangle Inequality, 1A1 P < AP I And lt lselcat
that all thc valucs I < d < I arc indccdobtalnablc ln laet, lor sue|
a c a hall hypcrbola olall polnts Q sueh that QA QP = d ls wcll
dcncd Polnts AandP arc loel ol thc hypcrbola )
$lneellnc APls parallclto thc r-axls, thlshallhypcrbolalntcrscetsthe
x- axls, ic , 1ls welldcncd
Froblem I
llndallrcalnunbcrsx |orwhleh
Solution I
lt lscasytoehcekthatr= 2 ls a solutlon Wc elalnthatl tlsthconly
onc ln |aet, c|v|dlngby i l on boths|dcsglvcs

. . .

.
i l
+
i l
+
i l
I
i l
Thclc!t hand s|dc |sa dcercaslnglunetlonol x and thc rlghthand slde
lsan lnercaslnglunet|onolr
Thcrclorc thclr graphsean havc at nost onc polvt ol lntcrscetlon

'

t'
r

'

ry

ro

Cott+:t:
\lorc
gcncrally.
:` -,a + I| - '

+(:- I|`
= ,a + I + I}` ,a -I-2|`
-,a + 2I)
|crc=I(2I+ I| IN
Pro11+t I0 Korean Mathemaries Comerition 200I[
letf N ? N N bca!unetlonsuehthai )(l.I| 2,
f(m -i . n) )n)! and f(m,
n I| = f(m, n)- n
|crallm, n N
lladallpalrs(,| sueh that )(.| .i
So1tio: I0
We havc
Therelorc
Le.
)(,| )( i |+ -I
)(-2|-;
2| + (-I)
)(I.|-
(

i )
( -|)
)(I
-I|-(-I)
2
)l,1}

I|
-
(

i)
. i
( " |)

( -I|
i
2 2
(-|( -I|= 2 ' i
N
cte that ilsaprlncnunbcrandthat- < -Ilor;N
We havc thc lollowlngtwoeascs:
I
/ = i and- i= l Ecncc ;= . and = i
2. / = 2 and -I= i Ecnec ; iiand =
s
3B 3. So1'Jot'o |t'rot'ory ro11,j
Thcrclorc(p. q) = , .,i) or , i i , ) .
Problem J? China J083|
Lct

bca|unetlondc|ncdon 1isuehthat

, ) =
' l) = 1 and
,a)-

,|) < a |
|orall a= |ln thc lntcrval i
Provc that

,a' , |' <

Soiution J?
Wc eonsldcr thclollowlneascs
J . ,a |, _ i}. Thcn
, a'
' |) < a- | _

as dcslrcd
2. , a |, > i }. By synnctry. wcnayassuncthat a> | Thcn
'a)
~
,|)

'a)
~ ' l) -')
'|)
as dcslrcd
Probiem J8
:
,a' , l ' -
')
' |'
< , a l' + ' |,
i a + |
= i- 'a - |)
< -
.
Flnd allpalrsollntccrs x, ) sueh that
x' ' = x - )

Soiution J8
*
$lnecr'

' = ,x+) x.
x- ) . a|lpalrsollntccrs,n -n) n E 7
arcsolutrens

$upposcthatx+ = Thco thc cquatlon bceoncs

" x-

= X + ,
ic
x

- ,+ l ) -

- Y =

5o

1
'
Jo

t '
o
1 t

'

ro

'o

_
aquadratlecquatlon lnx, wc ealeulatcthcdlserlnlnant
=

-?-1 !

-!= -3

-+i
Solving lor
ylclds
3* ..
<
3-..
3
-

3

Tausthcposslblcvalucslor arc i , and ., whleh lcadtothcsolutlons


(I, ) , ', i) , i , .' '.. i ) .and , . .)
Terefore, thclntccrsolut|onso l thccquatlonarcx, ) = , i , ) . , I ) .
(I, ) , , . i) , . .) ,and ,n, -n) . 'or alln E 7
Pro11+r J0 KoreanMathematics Cometitlon 200J|
Let
.
')
1

-.
for real nunbcrs x lvaluatc
So1tio: J0
Note thatf hasahal|-turnsynnctryabout polnt , i }.. l}.) Indccd,
. .
1
1

, i )
:
,
-
+ .

1 + .
1

=
1

+ .

from whleh lt lollowsthat

,)

, i x)
i
Thusthc dcs|rcd sunlscqualto i
Pro1l+r 20
Prove that lor n _ thccquatlon
l i i
- -
- - + = i
x]
x x
0= |ntccr solutlons
5o1 ut|o 20
Note that
1 i i 1 I

"
'
"+ "+ "
a ,.a) '.a) - ,.a) ,.a)

40
3. So1'1ot'o |t'xot'oxy xo11(,j

-
|ron whleh lt lollows that ll , , .

. :,)= ,, . . ,) lsanlote
gcr solutlon to
thcn
I I I
+ -

= I .

;
-

-
, , , -; ,-,-+, -+; -
-
+
)
= ; , , ,, , 2-,2-
.2-2- )
lsanlntcgcrsolutlonto
Thcrclorcwceaneonstruetthcsolutlonslnduetl vcly|lthcrcarcsolutlons
|or n = , . and 3

$|nec, I ls a solutlon lor n = I . 2, 2, 2, 2)ls a solutlon |or n = !,


and , 2. ?. 2, !, ! !, !) lsasolutlonlorn =
l t |scaytoehcek that ,2, 2, 2, 3, 3, ) and 2 ? ?. 3. ! !, I ?, I ?) arcsolu-
tlonslor n = andn = 3, .cspcetlvcly Thlsconplctcs thc proo|
ProbIem 2 AIME J088|
F|ndallpalrsollntcgcrs ,-, sueh thatthcpolynonlal
|sd| vlslblcby-

- I
SoIution 2J, AIternative J
Lct and bc thc roots ol -

- I = By Vieta's theorem,
= I and p = -I Xotc thatand nust also bc thc roots o|
:
,
'--

I = c Thus
:

-I and:
-

-I
Vul tlplylng thc |rst ol thcsc cquatlons by
thc sceond onc by

and us|ng thc laetthat = -I . wc|nd


:+-= -'
andq+- -
,

I )
Thus
.
'o |t'xot'oxy xo11(
],
5 o1'|ot
.
.

I
,+
)

-2)= I+2= 3,
,

-,

= 2= 7,
,,

,= ! 2= !.
't |ollowsthat: l l 1:=
..cl|n|nat|nga | n I ) g|vcs
- =

'

,'

+ ',+

,'

+'

,

P

= ,

,-,'+

, -,

,I
For 7 I , |ct/
-
-
-
Thcn /

3and /

= 7, and
P

-
+
+
2-
-
4
/
--

-
-

-
-
),

,-,

,
-

-
)
/
--
/
-
4
|or 7 3 Thcn /

= I 3, /, = !. /

, l23, /

= 322, /

= 3A 3,
k16 ~ ??0
..

-- ??0 3A 3+ 32? I23!- I3+ - 3+ I I


or

:-,= 3, -I)
8olution2J, Alternative 2
Theothcr |aetorlsodcgrcc I andwc wrltc
,c:

'
@

, - ,, ,-

- , =

--

I
Ccm
parl ngeocelcnts.
-
,

and |or 3_ /_ I. -
.

L
c,- ,= 0,c, = I
-c,- ,

= 2,
-.

.= c
42
3. So1'Jot'o |t'rot'ory ro1
lt lollows that lor k _ l , Ck = Fk+l ,the Fibonaeei number)
Thas a = Cl5 = Fl 6 3and b = -C14 - Cl5 =
-
F
1
7
= -I
7 0r
(a, b) = 3,-l ,
Comment. Conblnlng thc two ncthods. wc obtaln sonc lntcrcstlag
laetsaboutscqucnecsk2n andF2n- 1 . $|nec
lt lollows that F2n-1 and k2n satlsly thc sancrceurslvc rclatlon lt l,
casy toehcekthatk2 = FI + F3 and k4 = F3 + F5 .
Thcrclorc k2n F2n-1 + F2n+1 and
FrobIem 22 AIME J004|
Clvcnaposltlvclntcgcr n, lct,n) bcthcproduetolthcnon-zcrod|glts
o|n. l!n hason|yoncdlgl t, thcn,,n) lscqual to thatdlgl t ) Lct
5 , l
)+ ,( 2)+ .
+ ,)
Whatlsthc largcst prlnclaetor ol5!
SoIution 22
Consldcr eaehposltlvclntcgcr |cssthanI000tobcathrcc-dlgltnunbet
by prcxlng 0stonunbers wlth lcwcrthanthrccdlglts Thcsunolt|e
produetsolthe dlglts olall suchposltlvcnanbcrsls
0

0+ 00

l +
. . . +

) 0

0
= 0+ l+

+ )'- 0
Eowcvcr,, r) l sthc produetolnon-zcrodlgltsoln. Thcsunolthcse
produetsean beloundbyrcplaclng0by I |n thcabovccxprcsslon, slnee
lgnorlng0' slscqulva|cnttoth|nklngolthcnasl'slnthcproduets Xote
that thc nal 0 ln thc abovc cxprcss|on bceoncs a I and eonpcnsatcs
lor thc eontrlbutlonol000 alterl tls ehangcd to l I l )
Ecnee
5 = !'- l = !- I ; !

+ !+ l; = 3'

l03,
and thclargcstprlnclaetorl sl03
43
'
o1 t'r
o
t

'

@@@@@@
_ o '
.
I
23 Futnam J0?0[
yo1
em
cnec olnonzcrorcalnunbcrssueh that
L
t
x" be ascqu
Xn-2 Xn-1
Xn
2Xn-2 - Xn-l
r *
3, !,
| t b

and suelcnteondlt|onson Xl and X2 or Xn L c


Etablis h ncecssary
.
lorlnnl tcly nany valucs o|n.
a Integer
Soltion 23, Alternative J
We have
I 2Xn-2 - Xn-l
2

"
Xn Xn-2Xn-1
Xn-l Xn-2
- lf Thcn Yn - Yn-l = Yn-I - Yn-2 . l c . . Yn |s an arlthnctle
Let
'
ll

ls a nonzcro lntcgcr whcn ls ln an lnnltc sct 5, thc


suence. Xn
Yn's forn E 5 satl sly -I :: Yn :: I

S' e an arlthnctlescqucnec|sunboundcdunlcssthceonnond|crcnec

" 0|oralln whlch | n turn| npllcsthatXl = X2 a


W , Yn
Yn-
!
,
ncazerolntcgcr
Clmrly, thlseondltlonlsalsosuelcnt
So l tio: 23, Alternative 2
An eay lnduetlonshowsthat
X! X2
.
XIX2
Xn
(n " I ) XI - (n - 2)X2 - (Xl - x2) n +(2X2 - Xl ) '
for 7 ~ 3, !,
In t his lornwc scc that Xn wlll bcan |ntcgcr lor lnnltcly nanyvalucs
of 7 lf andonly l|Xl X2 |orsoncnonzcro lntcgcr .
Pro11+r 24
Slve thecquatlon
So lt|o: 24, Alternative J
It is

lear that X ;:: -2. Wcconsldcr thc lollowlngeascs


1.
?
>X :: 2. $cttlngX 2 eosa, 0_ :: , thccquatlon bceoncs
or
3 eos'a - eos a ,2(eos a + I )

2 eos3a =

!eos
44
3. So1Joto |txotoxy xo11m
-
lronwhlehlt |ollowsthateos= eos
.
Thcn - _a
2
= 2m7 m E 7 or+ * 2 E
,
- n7, n
$|nec0 7, thesolut|on|nthlseascl s
-
-
2 0 - 2 2
47 47
- eos - -= eos-
5
, and : = 2 eos-
7 '
2. - > 2. Thcn-
,
-- --
.
4) >0 and
.
- - x - 2 = ( .r - 2) (x + l 0
or
l t|ollows that
t' ->->,-+ 2.
Ecnecthcrc arc nosolutlonsl nthl seasc
Thcrc|orc -= 2, -= 2 eos47/5, and-= 2 eos47/7.
SoIut!on 24, AIternat!ve 2
lor->2, thcrc lsarcalnunbcrt >I sueh that
Thccquatlonbceoncs
| e
l e
6 1 1
t + -6 = t + -
t t '
(t
7
-1)(
t5 I ) = 0,
whlehhasnosolutlonslor t > I
|cneethcrcarcnosolut|onslor->2.
lor -2 _ : _ 2, plcascscc thcrstsolut|on
],
5o1'1oto |txotoxy xo11 m
.
45
Jrohlem
2 AIME J002|
For any scqucnecolrcalnunbcrs A = (, ,-,
.
} .dcncA tobc
thesequcnec (,- ,,- ,,- ,. . } .
5upposc that all o| thc tcrns o! thc scqucnec t( tA) arc I , and that
|
-,
O.
Find
-,
5oltion 2
5upposcthatthc rsttcrno|thcscqucnectA | sd.
Then
tA = {d, d + l , d + 2, . . . }
wlththc n
th
tcrnglvcn by d + (n - 1 ) .
Hence
A = {.., + d, ,+ d + (d + 1) . ILl + d (d + 1 ) + (d + 2) . .
wlththcn
th
tcrnglvcnby
1
,= .+ n l ) d + _ n I ) n - 2) .
Thlsshowsthat,lsaquadrat|epolynon|allnn wlthlcad|ngeocelcnt
1/2.
Since -,.= .. O. wc custhavc
1
,=

n- I) , - 92) ,
so -,~
(
1 - I) , I~ 92) /2 = 3I
ProbIem 20 Korean Mathemat!es Comet!t!on 2000|
Find allrcalnunbcrsJ sat|slylngt hccquatlon
5olt!on20
Settlag2x = and= b, thecquatlonbceoncs
I+
.
+ b
,
- - b - b O.
Nult|plylng both sldcs o| thc last cquat|on by 2 and eonplct|ng thc
|uarcs glvcs
I -
,
+ - )
,
+ (b :)
,
= O.
46 3. So1'|ot 'o |t' rot
Thcrc|orc I 2X 3x , and x 0 ls thc only solutlon
FrobIem 2? China I002|
Provcthat
SoIution 2?
Xotc that
2 ./ +1 -
,
,
2
<
-
I
/ + l +
_ _

Thcrc|orc
b
I
C
_
_
>
2
_
v- .
)
16.
k=! k=!
whlehprovcsthclowcr bound
0nthcot|crhand.
2 / . /"
1
2
>
-
I
_
+
_
Thcrcorc
b
1
b
_
_
<
1 + 2
_
/- .
)
2v - 1
<
1 7.
k=l k=
2
whleh provcs thc uppcrbound 0ur proo| ls eonplctc
Froblem 28 AHSME I000j
lctcrnlncthcnunbcrolordcrcdpalrso|lntcgcrs (m. n) lorw|lehmn ;
o and
SoIution 28
Xotcthat (m +n)3 m3 +n3 +3mn(r +n) . If +n " 33. then
333 (m +n) 3 m3 + n3 +3rnn(771 + n) " m3 + n3 + 99mn.
|cnec m + n - 33 ls a |aetor o|1713 + n3 +99rnn - 333. We havc
m3 +n3 +99mn " 333
(m +n - 33) (m2 +n2 - mn +33m +33n + 332 )
(m n - 33) [ (m - n)2 + 0+ll)

+(n +33)2 ) .
_ Q1'|ot'o |t' rot'ory P ro11
.
47
thcrcarc 35 solut|onsaltogcthcr. (0, 33) , I . 32) , " ' , ( 33, 0) , and
e0^
|
``
Qom
+:t
'
Vorc gcncrally, wc havc
a3 +b3
+
e3 - 3abe
(a +|+e) [ ( a - |)

+( b - e) 2 +(e - a) 2) .
2
yrohIem 20 Korean Mathematies Cometition 200I|
t
a, b, ande bc poslt|vc rcal nunbcrs sueh that +b e ! and
ab +be +
ea : -
Prove thatatlcast twoolthc lncqual|tlcs
re truc
Solt!o: 20
We havc
i.e.
i.e.
i.e.
Ie.
l a - bl 2, Ib - el 2, I c - al _ ?
(a +|+e) 2 16.
a2 +b2 +e2 +2( ab +be +ea) 16,
a2 +b2 + e2 - (ab be +ea) !,
(a - b)2 + , |- e) 2 +(e - a)2 3
and thcdcslrcdrcsult(ollows
ProbIm 30
"auatc

I
_
(n - /) ' , r+ /) .
k=O
4B 3. So1'|ot 'o |t'xot'oxy
SoIution 30
Lcts
-
dcootc thc dcslrcdsun Thco
l .-
.- _- , -+ ,

.-
.- _ -
I
-
.-

_
-) ! _ k
.- .
.-

. _ k
+
-
.

.
-
+
,
,-
.
i
.-,

.-
FrobIem31 Bomania 1083|
Lct0< :< 1. $olvc
lorposltlvc ounbcrs -
SoIution 31
Takloglog_ y|clds
:log

-=
,

CoosldcruoctlooslronP * P,
}, -:= : g -= logx /, x)= x
Thco both J aod g arc dcercaslog aod / ls loercaslog lt !ollows that
, -,g-, / -, has uolqucsolut|oox= :
FrobIem 32
What|sthceocelcoto-

whco
,+-: , +. -: +!x) +.
-
-)
lscxpaodcd`
5o1 a'
Jot'o |t'xot'oxy xo11m
J
.
9
oltion 32
Lt
_y~ :
-
,
+
:
-
-+

~:
-

-
-
-
= ( 1 +-: , +.-:

(+ .
-
-:
It iseY tosccthat:

,= aod
w havc
:
-

1 +.+
+,
!
.
--
- l
}
-

, -: , +.
-
-,
+
_
" : -+:
-

+

) l +.
-
-)
1 +
,
--.
tx+
:
--

+ .

.
-
-

:
--. .

+
.

n
=
.
-
ProbIem 33
:
--

+.
--=
.
--
1
+ .
--
.
-
:.

+. .+
=.
* .

+
_=
+
_
,, ,--
:
.+
3

--.
,
,--,
3
. _--
+
_

,
--
.
, .
- .
,
3
3
let m aod-bcdlstl oetposltlvc lotcgcrs
49
Find thcnaxlnunvaluco -
-
- -
-
whcrc x .sa real ounbcr |o thc
intervl t
5oIution 33
y synnctry. wc eao assunc thatm > - Lct , -
-
--

Since 0 < x < x" < -


-
aod0< ,< Thus
t- ,
-

-
-, ,- ,
-
,

-
---
:= ,
-
(
,
,
-
--
,
=
^pply|ogthe AM-GM inequaIityy|clds
s, l
,,
---
=

-

-

,
-
-,,
-
, ,
---

= -
,

-

-
+
m
^
, , ,:

---
-

=
n m -
=
-

,. " -:
---
_____
50
There!ore
3. So1Joto |txotoxy xo11m
~
Lqual|tyholds|!and only |l
or
m ),
=l ,
n

.
.

Comment. lor =

i . wehave
n
-
x
-
-

;
<


-
( n I )
-
*'
lorreal nunbers < < I Lqual|tyholds|landonly|l=n/ (n i)
FrobIem34
Provethatthepolynon|al
wherea, a


, a- are d|st|nct|ntegers. eannot be wr|ttenastheprod-
uct o! two aon-constant polynon|als w|th |nteger eoee|ents. | e , |t |s
|rredue|ble
SoIution 34
lor thesakeol contrad|et|on, suppose that
!')= '- a, ) ' a

' a
) |
|s not |rreduc|ble Let !')=') ) ) such that j)and ) ')are twe
polynon|alsw| th|ntegral eoeme|eatshav|ngdegreelessthan n. Then
')=j

))')
|sa polynon|al w|th|ntegral coec|entshav|ngdegreelessthan n.
S|nce
j'a
. ) )' a. ) =!' a. ) = l
andbothj'a, ) aad )'a. ) are |ntegers,
5o1'Joto |txotoxy xo11
J
m
a
d
Thus'
) hasat least n roots But deg < n, so')" 0 Then
j')=)') and
!')=
j
)

5
h|ch |npl|es that the lead|ng eoec|ent o!
!') nust be a negat|ve
,uger, wh|ch .s |nposs|ble. s|nce the lead|ngeoee|ent o!
!') |s i
yroblm 3
Find
all orderedpa|rsolrealnunbers
' ,) !or wh|eh
So lut|o: 3
' l) ' i
) ' i +' ) i,
and
' l,) ' i ,
' l,

) = l+

we coas|der thelollow|agcases
1. , = 0. Thea |t |sclear that = , = and '. ,) = (0, 0) |s a
solut|on
2. ,< 0 Bythesynnetry. wecan assune that > 0 > , Then
(l+) ' l+
) ' l+ ) > l and l+,< l Therearenosolut|ons
|ath|scase
3. ., > aad = , By the synnetry. we can assune that >
,> 0. Thea
' i ) ' l
) ' i) > i :> i, .
show| ag that there are aosolut|ons|n th|s case
4. .,< 0aad= ,

Bythesynnetry,wecanassunethat< ,<
0. Hult|ply|agby i and i ,the rstandthesecondequat|oa,
repect|vely, thesystenaowreads
l - " = , l ,) ' i ) = i - , ,
I -
s
= ' i ) ' i ,) = i , ,
Subtract|agtherst equat|on lronthesecondy|elds

-,
='

':-,)-,' -, ) ' l )
S|ace<
.
,< 0, - ,> 0, ,< 0,
- < 0, -,< 0, aad

, > 0. Therelore, the lelt-hand s|de ol' i ) |spos|t|ve wh|le


therlght-hands|deol' i ) |saegat|ve
Thusthereareao solut|ons |a th|scase
52 3. So1'Jot'o |t' tot c'oty to11(q
-
5. r=y.
Then solv|ng
- X
8
= - r'
7
xy
7 = rX7 " x8
leadstor= 0, 1 g g wh|eh|npl|esthat ,r y) (0, 0) or - | -1 ).
Thereore, (r, y) = (0, 0) and , l, are the only solut|ons to the
systen
FrobIem 30
$olve theequat|on
lorreal nunbersr

SoIution 30
Rearrang|ngternsby powersol2 y|elds
2
X2
X + 2
-
+

r i
- 2(x
2
x - I) -0 , ,
$ett|ngy ~ x
2
- andd|v|d|ngby,ontheboth s|des, ,,beeones
or
x(2Y - l )+ y(2X := 0. (2)
$|neefr) 2x ~ and x alwayshave thesanes|gn,
x(2Y - 1 )

y(2X ,_0
Eeuce |theterns oa the lelt-hand s|de o, ,,are nonzero, they nust
have the sane s|ga, wh|ch |a tura |npl|esthat the|r sun|saotequal to
0.
Thereore(2) |strue|laadonl y|lx = ory = 0, wh|chleadstosolut|ons
r--1 , 0, aad l
FrobIem 3?
Let .be an |rrat|oaalnunberand let be an |ntegergreaterthanI
Provethat

. . )
|san |rrat|oaalnunber
],
5o1'|ot'o |t'tot c'o ty to11( m
.
5olution 3?

t
s0dlet

- . )
.

53
lor the sake ol eontrad|et|on assune that |s
Thea
.
ratloaal Thenby us|ngthe |dent|ty


repeatedlyor m= , , we obta|nthat

|srat|onallor all
m C ^
lapart|cular,

= -

- ..

is ratlonal, |n coatrad|ct|on w|ththe hypothes|s


Thereloreourassunpt|oa|swrongand|s |rrat|onal
ProbIem38
Sclvethesystenoequat|ons
( Xl - X2 + X
3
)
2
= X
2
(X4 X5 - X
2
)
(X2 - X
3
+ X4 )
2
= X
3
(X5 xl - X3 )
(X
3
- X4 X5)
2
= X4(XI X2 - X4)
(X4 - X5 + XI )
2
X5( X2
+ X
3
- X5)
(X5 - XI + X2
)
2
- XI (X
3
+ X4 - xd
|crreal aunbersXl , X2
, X
3
, X4 , X5'
5olution 38
Lt Xk+5 = Xk . Add|ngtheveequat|ons glves
5 5
_-- 4XkXk+l + 2XkXk+2) = _-x 2XkXk+2 ) '
k=l
k=l
It|ollows that
5
_-- XkXk+d 0.
k=l

54 J. So1Joto |txotoxy xo11 m


\lult|plylng both sldes by.and conplet|ng the squares ylelds

_. :-., ) 0,
-=,
*
ron whlch :, = :

= :, = :

= -, Therelore the solutions to Ihe


systen are
|oraE R
FrobIem30
Let , andbeconplexaunberssuchthat, =.. +,
l and ,=1
Lvaluate
SoIution 30
i l t
+ +
,+- t , l , i
Let5 bethe deslredval ue Xotethat
Llkewlse.
and
Eence
5
But
,+ - i =,+I : Y = ' i ) ,, | )
,+ l = ,,- l ) ' i )
,- i =' i ) ,- i )
l i 1
,- l ) ,, t
+
,, l ) ' - l )

' i ) ,
l )
+ , + l i
,- | ) ,, i ), i )
=
,
-

l
),

),

- l )
-l
,- ,,,+) ++,- i
l
,, + , + )
.',+,+ )= ,+,) ',
)= i
Therelore5 =.}
5o1 a'1oto |txotoxy xo11 55

JJohIm
40 USSB I000|

Patlsgolngtop|ck threenoa-zerorealnunbersandVr Ta||sgoag


r

aagethethreeaunbersasthecoeelentsolaquadratleequatlon
t
srr
.
_+__=o.
r. Pat w|ns the gae |l and only l the resultlng equatlon has two
d|stlactratlonal solut|ons
Wo hasawlnningstrategy
5ol
ution 40
Jr.Pathasthew|an|ngstrategy Asetolthreedlstlnetratlonalnoazero
au
bersa |. and suehthata+|+c=0, wllldothetrle LetA, B,
d Cbeaayarrangenentola, | and andlet !,)=A 8+C
Jhen
!, l )=A PC =a|=0,
whlch|npllesthat I lsasolutlon
5lacetheproductolthetwosolutlonslsC }A, theothersolutlon|sC}A.
d lt lsdlerentlroni
ProbIem 4I USAMO I0?8|
iven that the real nunbers a.| c.1. aad e sat|sly s|nultaneously the
telatlons
a+|c+1+e 3and a+|1e= i 1.
deternlae the naxlnun and the nln|nunvalueola

8olution 4I, AIternative I


S|ace the total ol|. e 1. and e ls 3 a, thelr average ls = 3 a) }1
Lt
| = |, , c = : , , 1 = 1, , e = + e,

Thea|,+ c,+1, + e, =0 aad


, l )

0 _ a i1a=a,a i 1)
Therelore 0 _ a _ i 1} w|ere a= 0 lland only .l|= c =1= e=
=4c=l1}lland only ll |=c=1=e=1}

l
. . . ,,
56 3. So1|oto |txot
Solut!on 4I, AIternat!ve 2
By the BMS-AM !nequaIity, t , o|lowsron
|

+ c
+ )
+
_

'| + c + ) |
)

1
and the rest olthe solutlon ls the sane
ProbIem 42
llnd thereal zerosothepolynonlal
wherealsaglvenreal nunber
SoIut!on 42
Wehave
' a)
1
'

+i , '

!r+i ) a

=0
llvldlng by ylelds
By settlng,=+l }, thelastequatlonbeeones
,

!, a=0
lt ollowsthat
+- = i ,

whlch la turalnplles that, lla 0, then the polynonlalP, ') hasthe


realzeros
l+ 2,- 2
~~= = -

-
2

lnaddlt|oa, lla , thea P'-) alsohas the real zeros


FrobIe 43
Prove that
l , a 2,- 2
2
i l 2n i |
- -

2 1 2n
v
lorallpos|tlvelntegersn
xo11
g
o
[ution 43
eastrongerstatenent
We
prov
i l !n i i
2


,ln+1
.
We use laductlon
l theresultlsevldent
[r


th statenent lstruelor sonepos|t|veuteger k, | e .
Suppose
e
i l 2/- 1 i
_

,l/ I

Then
| l 2/ i !/+i i !/+i
_

!/+ 2

,l/+i

!/+!
l0or3erlorthe|nductlon step topass |tsuces to prove that
This re3ucesto
i.e.
i 2/+1 1
,l/+i

2/+2

,l/+1

2/+l

l/+l
.

!/+! l/+1
'1/+1/+ | ) l/+1) '1|

+/+1) 'l/+i )
i.e.
0 < /.
wh|ch| sev|deat 0urprooll seonplete
57
Coment. By us|ag $t|rllng nunbers, the upperbound can be ln-
proved to l}lor suc|entlylarge n
Problcm 44 USAMO FroosaI, GeraId Heuer|
Ut
P')=a,

+a, :
-
+


+a
aoazeropolynonlalw|th lateger coeelents such that
P'r) =P' s) =0
for oelategersr and s wlth 0 r s
Prove thata,_ -sorsone /
5B
3. So1|ot o |t'xot
Solution
Wrter =, - aod
, =,
-
+

-

+

+
-

here ,f O. S|oc

hasa posltlve root. by lescartes' rule osla,


e|thertherenustex|st sone / !orwhlchbk > 0
>
or > 0
'
- k+l ,
-

lthere exlstsa k lorwhlch k > 0 2 bk+1 , theo


ak+l -sbk +bk+1 -
l,> 0, theaa
-
=-
-
-
loelthercase, therel sak such thatak _ - asdeslred
Problem 4
Letn beaglveorealounber llodallconplexounberssuch that
Solution 4

:
+

:
,

=n
+n
Conpletlogthesquareglves
l e

X x
,

:
X
+
t
+
- ,
=
" |
+n

+n,

_
- t

:
+n n
$

/
ettagY =z , - : , theaboveequatloo becones
l e
Thus
,
" Y n
+ n) =0,
,- n
~ i ,,+n) =O.
2X
2
z

=-n or ~
=n+:
- ,
i `
whlch leads tosolut|oas
=
z
l f -2 aod=_|ln= I .
1
5o1'1oto |txot'oxy xo11+
m
_
o
l0

The
se
ueoceglveo byXo =Xl =aod
i percdic
Provethat a=|
5olution

Xn+l = n-
+
,
: Xn
59
Mult|plylogby:,oobothsldesotheglveo recurslverelat|oo y|elds
2XnXn+l
Xn-l Xn -t
or
2(XnXn+l - : , Xn-l Xn - :
[t Yn ~ Xn-l Xn - :or n N. $loceYn+1 Yn/2, {Yn} l s ageonetrlc
gueace llXn |sperlod|c. theo so lsYn, whlch .nplles that Yn ~ 0 !or
ml C N. Thereore
=XOXI =Yl +t=t
ProbIem 4?
Lt a, aod-bereal ounberssuchthat
(a +- : , c+ -
- , > +-- :,

}
rove that
5olution 4?
For thesakeolthecootradl ctloo, suppose that ooeol+or +-
less thaa or equal to : $loce ,+ -- : , 2 0, + :aod
+ d
2
- tnust have thesaneslgo Thusboth+ aod+ -are
I
=thao : Let
=:- - aodY :- .
-
hea0 <Y _ : Vult|plylogby!on bothsldesotheglveoloequal|ty
gve
, >
:+:-- : =,,- :- :-,

++X +
+-+ y - :- :-,
a - .

-,

-,
-X + ,,

_ , + ,, =
+ z, + ,

60 3. So1|oto |txot coxy Pxo11+ m


~
or 0> - :+
=- ,, wh|eh |s|aposs|blc
Thusourassunpt|on |s wronand both+b2 and +a arcrcate
than :
Problem ;s
l|ndall eonplcx nunbcrs = sueh that
:+

, ,: , , + :, ::+I) =:
Solut!on ;s
Xotc that
s:+:, +:,
+
. , : :: :,=s
| c
:-+s, :+, :+, :+:,=:s
$ctt|n= :-+
,
and .=y|clds
- +, , +: , - : , ,- :, =s
| c

- i , - +,=:s
| c
. ,a.- :

+=0,
| c
,.- ++, .+ ::, " o.
Thcrcorc thcsolutions|othcglvcn oquation arc
Problem ;0
= `
-

and" '-
,
: :
Lct -, -

-
--
, bc thc zcros dicrcnt ron I o thc polynon|al
t,=
-
- : :
Provc that
"^+ "
|
^+ . t
|
I - -, :- -

:
-
-
-.
- :
=
.
:
5o1|oto |txot coxy Pxo1 | +
],

5olution ;0, Alternat!ve I


)or i : : lct ,= :- Lct
Then
P l - , : - ,
-
- |
,= =
- -

,, =, -: ,
-

--
'+, :,
--

-
+

a
n3a
iS arcthcnonzcrorootsolthcpolynon|al -, as
,
, I - ,,
-
- t - t
0

=
=. ,

: - -

6
Thus thcdcs|rcdsun|sthcsuno| thcrce|proealso|thc rootso|poly-
n
omial that |s.
I :
-
+ ~+
+

:-

I - -

I - -
--.
: : :
- - + +
+
.

--.

,+.
,

-
+

+,.


--
,
,


-
ythe Vieta's Theorem, thc rat|obctwccn
and
!=,
-
s equal to thcadd|t|vc |nvcrsc o|thc rat|obctwccn the eoce|cnt ol-
and thceonstanttcrn |n , | c . thc dcs|rcd valuc |s cqual to

n - :
j
=

,
=

a 3cs|rcd
5olut!on ;0, Alternat!ve 2
For anypolynon|alu, , o|dcrcc-: whosczcrosarc-, -


-
--
,
the lollow|n|dcnt|ty holds
t
|
u -

+
,
+
=
- - -

- ~ -

- - -
--
, u ,

_
_1
_
'
_
o

t
_
_
'
_
o
_1_
t
_
'r_
o
__

_
'
_
o
r _r

lor
P, )

l
: -
l = : - .- -
l
P, l) =r and
r,r~ l)
P i) = r l)- n ~ 2)-

- 1

It lollows that

l
-
l
-

. .
-
l P', l)
=
n 1
l ~ :

l r

l - .,
=
/, l) 2
FrobIem 0
Lct aand!bc glvcn rcalnunbcrs
$olvcthcsystcno!cquatlons
- ,

l -,
a.
,
!
i -,
!orreal nunbcrs and,
SoIution 0
Lcto=-y andr=~ y. Thcn

- r o - r
0 < - , =r< l = "
2
andy = "
2

Addngthc twocquatlonsandsubtraetlngthctwocquat|onslnthcorlg-
lnal systcn ylclds thcncwsystcm
o~ o, = ,a!) .l or
ro, = ,a !) .l- or
\lultlply|ngthcabovctwocquat|onsylclds
or, l or) ,a

) , l ~ r) .
hcnecor=a
~
!

ltlollows that
,a - !) .l a - ! , a !) ;i a - !
o= and


1 . a ! |-a- !
whleh ln turn lnpllcsthat

a-!.a ! !-a.a- !
,

,)
-
.l a - !

.i" a-!

whcncvcr0 < a ! < l

J
'\U1T'
/N
/T\t !1\tN'
,=-.
.\. .
`
, | i

.
`
U ! | / |

,
X .
. 5LUJN5
AVANLL !LLN5
Jrohlcm
I
8valuatc

5olution I
Lt
.
, x,= , -)

,
.
,
Lt , -I+.

Thcn
_3
= I and
_
t U +=

Ecnec
+
,

,,


+
' '
= , l )
_
'
_
)+
_
,
_

)
= .
+ _
,

+
_
)

, l
_

=
.
+ _
(
_
)

a
+
_
(.)

= .

_
.


Thusthcdcslrcd valuc ls
.

=
+
ProbIcm 2
Lctx, y, z bcposltlvcrcal nunbcrs sueh that x
4
z4 = l
0ctcrn|ncwlthproolthcnlnlnunvalucol
8olution2
x
3
y
3
z
3
+ + -
I x8 l ,

For 0 < o < lct

,o,~ o, o, Lct ~bc a posltlvc rcal nunbcr


ythcAM-GMincquality,

,
, , ~ ~o , o,

, o , _

~o

o,
,
66 4. So1'Jot'o Attt xo1 | _

~
$ettlngA =sln the above lnequalltyy|elds
s,, ,

'
or
lt lollows that


-+ +
:- ' :- ' t- '
wlth equalltyllandonly ll
s
),
=
- ' '
+ +
, : ' , :- ' , :- '
,'+)'+ ' ,v
s
v
s
t
- =

==
v
Comment Thls lsaslnple appllcatlonolthe resultolproblenllla
theprevlouschapter
Froblem 3 Bomania I000|
llndallreal solutlonstotheequatlon
.

+l

+1

=

Solution 3
lor < 0, the lunctlon ),,= .

+l +1 ls lncreaslng, so the
equatlon),,=0has the unlquesolutlon =I
Assune thatthere ls asolutlon 8 a Then
8
2
=!+l + 1 l
so 8 ,. andhence 8] :
But then 8 8 ] ylelds
. . =, :+ l ) l+8, 8
,
whlch ln turn lnpllesthat
1 > 1 ='.
)

8
2
.
o| 'Jot'o Attt xo11

25 + l+
1 > 8
2
, acontradlctlon
>
h

!ore- - : ls the only solutiontot eequat|on


[crc
.
yroblem 4
|t

--_, beasequencesuch that


, =.and
laralln E N.
-
1
-+:
=

.
+ -
-
F|ndanexpllcltlornulalor
-
olut!on 4
3clvlngtheequatlon
:
= - - -
.
lsto=+. Xote that
-+
,+.

,
+ 2v-+.
=
-+.

-+
, .

2v-+2 - .
Thereore,
and
Problem
.

.+ t ,

+t,
-
=

v+: ,
2"
- t
Letx, y, and Z beposltlvereal nunbers Provethat

.
,
=
, + +

+ , +
8olution
Not that

+
_ t
+

, + , +

+ ,+ v
+

67
6 |!|ot!A t t t xo1

In laet, squarlngboth sldes ol t heabove lnequallty ylelds


whleh lsevldentbythe AM-GM inequality Thus

=
.
,+, +,
-
+
,
+
v .-;+.

llkewlse,
and

;
+ ,
+ + -
-
v+ ,+
.

.
+ , + +
-
.+
,
+
.
Addlngthelastthreelnequalltlesleads to thedeslred result
Froblem 0
llnd, wlthprool,allnonzeropolynonlals, , suehthat
, , +,, , + l ) =o.
Solut!on 0
Let , , .
-
:, , , where and are non-negatlve lntegers
and
, ~ - .
-

, - ,
, = and Zi = t or i = : : ./ Theglveneondltlonbeeones
.
-
t ,+: , - , -

,- ,
~ .
-
+: ,
-
- : , .~ , -

,
,- , ,
+:- , +l-

,

+ :- , ,
Thus .= -. and ,ls nonzero. so .~ -: $lnee,= l , :- ,= 0
Then
-
~
-
thatls, =
Thus,lsolthelorn

-
- : ,
-
, ,
llvldlngby
-
- :, +t, thelastequatlon beeones
, ,~ , , , +: ,

| !|ot! Attt xo1 |


69
ye
clal that, , ~ : $upposc not thenelearly 9 nusthaveatleast
oaecou
plex root-= 0. Now
,- , , -, ,,-+: , ~
,

-
'

,-' , 0.
adso on
3lnce9 eannothavelnnltelynanyroots allltsrootsnusthaveabsolute
value :
Now,
sc -- t ,

:
Clearly, if
then
, -- : , ,=,-- : ,,-,
1 + ,i :- ,i
-E
: :

@ut-|salsoa root o! ,sothesaneshouldbe trueo!-

:+,i :- ,i
- E
: :

Thls|s absurd Eenee,9 eannothaveanyroots. and , , ~ :


Therelore,t he, areallthepolynonlalsol the!orn~
-
- : ,
-
lor
m E N.
Problem ?
Let,, N N bealunetlonsuehthat,,+: , > ,,and, ), ,=+
crall
valuate,::
5olution ?, Alternative I
We prove the lollowlnglenna
!mma lor = : : ,
1. , + , : + and
2. , : +,
_
+

o1'Jot'oA t t t xo1 1

Piuo] Wc usclnduetlon
o

notchat , l ) = l. othcrwlsc3 ' , l ) ) , l ) =l h


rs rnpossrblc $rnec N N , l ) > l $

w
iej


rnec ,o- l) > f ,
|s |nercaslng Thus l < , l ) < , , l ) ) = 3 , l)

.
H _
,.) =,, l ) )=3.
or - Ecnee
$upposc thatlor soncposltlvc l ntcgcro_ l
Thcn,
and
,. 3n
+l
)
=
(
J
(3n
+l
) )

3n
+
2
,
asdcslrcd Thls eonplctcs thc lnduetlon 0
T
,
hcrc

rc 3n l l ntcgcrs m sueh that 3n < m < . 3n and th


3 l rntcgcrsm'sueh that
crcarc
f (3n
)
. 3n <
I
< 3n
+l
,. 3n
)
.
$lnec] ls an lnercaslnglunetlon,
f (3n -) .3n -m.
!or_ _ 3n. Thcrclorc
,. 3n -m) ,(3n + m))=3 ( 3n -m)
lor _ m _3n . Ecnec
,.1l ) .. 3
6
+ .543
)
=3
(
3
6
-.543
)
=+3I ,
SoIution?,AIternative 2
lor lntcgcr o lct o
.
d .

a, a

a, cnotc thc basc 3 rcprcscntatlonol


o
Lslngs|nllar lnduetlonsas lnthcrst solutlon h , wc eanprovc t at
, o)

=
.a

a, ila
:
l.
la

a, lla, .
$lnec.l

=...l. ,.l )

= l..lcor
, .l ) l 3
2
-. l

-.
l+l 3
7
33l

| 'Jot'o Attt xo11


7
yrohIem
8 China I000|
]et

bc thcsctolallpolynon|als ,) wlth lntcgcrs eocelcntssueh


tbat
,)" lhasatlcast oncl ntcgcr root
For caehlntcgcr/> l,ndm, thclcastlntcgcrgrcatcrthanllorwhleh
therccxlstsanE F suehthat,)=m,hascxaetly/dlstlnetlntcgcr
;oats.
5oIution 8
Supposcthat E F satlscsthc eondltlonthat ,)=m,hascxaetly
j1lstlnetlntcgcrroots, and lctabcan. ntcgcr sue| that, ,a) l Lct
gk bcthcpolynonlal lnF sue| that
, ,-)=,-a)
for all
baw, ,)=,a) l . sothc eonstanttcrno|, lsl Xow, ,-)=m,
bas cxaetly/dlstlnetl ntcgcrrootsr , , r

e a , ., . sowc eanwrltc
whcrc))lsanl ntcgcrpolynon|al
bate that r,r

r, dlvldcs thc eonstant tcrn o| , ,-) - m, whleh


euals l m,

S|aecm, > l . l m, eannotbc.


bcwr
l
, r

. r,arc dlstlnetlntcgcrs,andnoncolthcn |s. so


heaec
m,_ /}.] ' /}. '+l
Thlsvaluc olm, l s attalncd by
Thus,
, , -) = ,
l )
,

)
, | ) ,-l ) , .) ,-.)

- , l )
,
/}. ) + /}.] ' `/}. ' + l
m, = /}.] ' `/}. ' + l
72
..~.~. o1't'oA t t t xo1 1

FrobIem 0
Lct Xl .and
lor H I
Provcthat
Xn+l =X- Xn + I .
I -
_1_ I I l l
.
2
,- 1 < - - -

+ - < l
-
Solution 0
$lnecXl .and
Xn lslnercaslng
ThcnXn I = a
Ecnec
Xl X2 Xn 22n

I
Xn+l - l
or
I I
=

-
Xn Xn - I Xn+l l
.
whleh lnpllcsthat

. . + - = l -
I
Xl X2 Xn Xn+l - I

Thuslt suecs toprovc that, lor n E N,


or
1
I
I
.

< I
- < I
Xn+l - I 2


. <
Xn+1 - 1
< .2n
Wcusclnduetlontoprovc I )
I)
lor n =I X2 - X
2
+ I + d )
,
-
I
X I = an ( l bceoncs 2 < +< ! h h

truc.
w IC lO
Xowsupposc that I ) lstruclorsoncposltlvclntcgcr =k, l c ,
2
k -1
k
. <
Xk+l 1 < .
2

Thcn lor n k+ I , t|clowcr boundol I ) lollows |ron


2)
'1ot'o Attt xo11
73
SlaecXk
+
l lsan lntcgcr, thclowcr boundo| 2) lnpllcs that
lrom
wh.eh it lollowsthat
a dcslrcd
[hlsnlshcsthclnduetlonandwcarcdonc
Problem 00 IranI00?|
Supposc that !
: j
+
j
+
ls a dcercaslng |unetlon sueh that |or all
x, Y E j
+
,
!,:+,)!,!' )!',) ) =
!' !'+!' ,) ) +!',+!'
) ) )
Prcvc that
!' ') )
5olution 00
Settlng, X glvcs
!,.) + !,.!') ) !'.!+!) ) )
RcplaelngX wlth
!')ylclds
!'.!' ) ) +
!'.
!, !') ) ) !'.!' !') +
!' ') ) ) )
Subtraetlngthcsc twocquatlonsglvcs
!' .!' ') ) ) - !' .)
!'.!'')+!'' ) ) ) ) !'.!
+
!') ) )
|l
!''))> thclclt handsldco|thlscquatlonl s ncgatlvc, so
!!' )+
!''
`
)
>
!'-!'))
nd
!,)-!' ') ) < X t !') ,
eontradletlon Aslnllareontradletlonoeeurs . l!' ' ) )
<
Thus
!',) ) X dcslrcd
Comment. lnthcorlglnal lornulatlon
!
was ncant to bcaeontlnous
|unetlon Thcsolutlonabovcshowsthat thlseondltlon lsnotncecssary
1
74 4. So1' lot'o Attt xo11

Froblem 0I Nordie Conteat I008]


llndalllunetlons Q Q sueh that
',)+ ' - ,)=.')+ .',)
lorall , ,E Q
Solut!on 0I
Theonlsuehlunetlonsare, )=/lorratlona|/ Anysuehlunetlca
works, snee
,+,)+ , ,) =/,+ ,)

+ /, ,)
=/
- ./,+ /,

./,+ /,

=./+ ./,
=.')-.',)
Xowsuppose lsanylunetlonsatlslylng
'-,) -'- ,)=.')-.',
Thenlettlng=,=glves. ') =1'
' so ,)=
We wlllprove by lnduetlonthat,o)=o,) loranyposltlvelnteger
oandanyratlonal nunber
Theelalnholdsor o= and o = l . leto _ . and supposetheelalm
holdsloro I and H .
Thenlettlng= 'o- l ) .,= .n :heglvenequatlonwe obtaln
so
' o)+ '
o
-
.) ) =' 'o
~
l) )+ ' 'o- l )- )
=.' ' o l ) ) -.' )
,o)=., ,o
l )) + .,) , ,o- .) )
=.'o- l )

, + .') 'o .)

')
= '.o
-
1o+ .+ . o
+ 1o 1) ' )
=o
' )
andtheelalnholdsby lnduetlon
Lettlngx =l ntheglvenequatlonglves
!',)+ '
,) =.
!,)+ .',)=.',)
so , -,) =
!,,)lor all ratlonal ,. thus ,o) =o, ) lor all lntegers
H

| 'Jot'o Attt xo11


[ow
let/=, l ) . then|oranyratlonal nunber =j}),
)

),)=,)`
='j)=r

' l ) =/j

b
[husthelunetlons')=/ . /E Q. are the only solutlons
PJoble 02 KoreanMathemat!es Cometition 2000|
]et al
Provethat theequatlon

,- l )= ' a) ' .+ a)
75
halour dlstlnet real solutlonsandnd :hesesolutlonsln expllelt lorn
SoIution 02
]ookattheglvenequatlonasaquadratleequatlonlna
a

-la+ .-

=
Thedlserlnlnanto|thls equatlonls

-
1

+ 1

= ,-.-

Thus
l+ '+ .

)
a =
.
Thersteholeea=+yleldsthequadratleequatlon s a=.
whosesolutlons are
, l+v
"
.

The
seeondeholeea=. ylelds thequadratleequatlon

+ .+ a=.
whcsesolutlonsare
I+,
Thelnequalltles
1 ,l+ 1a 1 + ,l+ 1a
-;
, - ,
.

.
hcwthat theloursolutlonsaredlstlnet
76
lndccd
rcduecs to
4. So1'Jot'o Attt xo11m

l+,<
l

.
.. <l ,l-1a
whleh|scqulvalcnt to
1,l-1a<1-a,
or la<1a , whleh lscvldcnt
Froblem 03 Tournament oTowns I00?|
Lct a, | ande bcposl tlvcrcalnunbcrssuehthata|c=
Provcthat

l l
a - | - l
-
| - c + l
-
c - a + l
i
Solution 03, Alternative I
$ctt|ng~ a-|.,= |+cand e+a. thclncquall:ybceoncs
l c
i . e.
l c
| c
| c
l c
i l l
- ~- - < l
+ l , + l + l

'
i l
- -< ~
, - l - l
~
- l
, - + .
-- < ~
,, +l ) , + l ) + l
'
,++.-,+ -. _ ,+,+-
+,--._ ,
., a-|+c) + . _ ,a+ |) , |-c) ,e+ a)
., a-|-c) _a

|+ a|

-|

c+|c - c
a-ca

By thcAM-GM inequality,
,a

|+ ae+l ) _l.a|c la
}lkevlsc,
=
1
)herelorcwc only nccd toprovc that
., a+ |-c)-l l,a-|+e) ,
|.e.
l a+ |+e
whlch|scvldcntlronAM-GM inequal\ty anda|e= l
5olution 03, Alternative 2
let a = a .|~ | e= e Thcn a, |, c, = l Xotc that
a-| a|, a, | a, - |,
,a |)
<

whlchlnpllcsthat
Therelorc,
L|kewlsc,
ad
l
1

a-|-l a+|+a,|,Cl

l
<
a
i
|-c+l a, + |,+e,
l
|,
<
L -a+l a,-|i-Cl
^11lagthc thrcc lncqualltlcsylcldsthc dcslrcd rcsult
77
1
J
7 o|ut |oo s toAdvaoce dPob|e

Froblem 04 AIME I088]


llnd all |unetlons ), dcncdonthcsc: olordcrcdpalrsolposltlvcl ntc
gcrs, satlslylng thclollowlngpropcrtlcs.
) x, x) J: ) x, q)
) q
x) ,x -q) ) x. q)= v) x, x+ )
SoIut!on04
Wc elaln that )xq) lenx. q) thc lcast eonnon nultlplc olJ anc
q lt is elcar that
len, x, x) x
and
lenx, ) lenq,x)
Xotcthat
xq
lenx. ) =
d ) ge x. q
and
ged x. q) =ged x, x+q) ,
whcrcged u L) dcnotcsthcgrcatcsteonnonclvlsor olu andL. Thcn
x-q)lcm x, q) = xq)
xq
ged x, q)
xx + q)
^ q
ged x. x+ q)
lenxx+q)
Xowwc provcthat thcrcl sonlyonc|unetlonsatlslylngthceondltlonsol
thcproblcn
lor thc sakc ol eontradletlon. assunc that thcrc ls anothcr |unetlon
x, q) alsosatlslylng thcglvcn condltlons
Lct5 bcthcsctoallpa|rsolposltlvcl ntcgcrs x, q)suehthat], x, )=
x. q) . andlct ,r n) bcsuehapalrw| th nlnlnal sunr+n It lselcar
that r = n. othcrwlsc
)r

=
) rr) ~ 771 ' rr) r n)
Bysynnctry ) x, g) = ) q, J , wc ean assuncthat H 771 > a
Xotc that
n)r, n r) = r + n ~ r) [ }r n - ^`
= n- r)
)r,H + n - r) )
= ,n- r)) m,H,
4. o|ut|oosto Advaoced Prob|ems
a

_ljcw|sc
M
),r n- m) = )'r n)
.
n
- !
r, n- ,
n
r. H .
Slnec )rn) / "
.n) ) r n- ) = , r.n- r)
[hus rn- m) E 5
79
@t rn - r) hasasnallcrsun rn- , , aeontradletlon
|hcrc|orc our assunptlon ls wrong and )
x q) = |enr,) ls thc only
solutlon
Problem 0 Boman!a I000[
Consldcr H ecnplcx nunbcrsZk . suehthat I Zk l _ l k = 1. 2, . .n
Provc that thcrccxlst C1 , C2, Cn E { - 1. l } suehthat, |or any _ n,
I C1 Z1 +C2Z2 +

+c,:, .
oIut!on 0
Callahnltcscqucneco|eonplcxnunbcrscaehw|thabsolutcva|uc not
cxeccd|ng1 arccnscucncc
Cal|agrccnscqucnec {zdk=
l
haq l| it has aricnd scqucnee {ck
}k=l
clsand - Is satls|y|ng thc eondlt|on o|thcprob.cn
Wc wlllprovcbylnduetlononn tha:allgrccn scqucnecsarchappy
lor ~ 2, th|selalm lsobvlously truc
Sapposc thlselalnlstrucwhcn H cqua|ssonc nanbcr711. lor thceasc
cln= H l . thlnk o|thc Zk as polnts ln thceonp|cxplanc
lor caeh / lct |, bc thc llnc through thc orlgln and the po|nt eorrc-
s,ondlng t

Zk . Anong thc |lncs / .2 . /

, sonc two arc w|thln 1 ol


caeh othcr supposcthcyarc ande{: w|th thclcltovcr onc bc|ng/.
Thc |aet that / and /_ arc w|thjn 11 olcaeh othcr |np|lcs that thcrc
cxlstssonc nunbcrC{ E { I i[ sueh that <' :+e{z{ has absolute
valacat nost l .
bow thc scqucnec Z' , Z
l
' Z4 , Z5 , . . . , Zk-1 lsa |-tcrn grccn scqucnec so,
ty thc lnduetlonhvpothcsls. lt nustbchappy. lct c' , c- . C4 ' C5 - Ck+ 1
tclts|rlcnd
lct
' ~ I
'|cn thc scqucnec { Ct } 71
1
ls thc |rlcnd ol {z, } _ lnduetlon ls now
ccaplctc
l
B0 4. 5o|ut|oos to Advaoced Prob|e

FrohIem 00 ABML I00?|


llndatrlpleol ratlonalnunbers a, b, e) sueh that
SoIution00
Letx= ;; l and y = ; Then,

= 2 and x= ,, l Xotethat
l =,

l= ', l ) ',

-,- l ) .
and

l
l,
+
l,
+
l
, + , + =
l
,

-l,-l,-l
l
whleh lnpllesthat
~
+
= " i=

l

,-,-l
or
v
~ = -
, - i
0ntheotherhand,
l
,-l)

l= ,

+l = ',+l ) ,

Y -l )
lron whleh ltlollowsthat
, + l
=
=
, + | l
Conblnln' l ) and 2) , weobtaln
Consequently,
i sadesiredtriple
',+ l)

l
' l)
,.)
o|ut|oosto Advaoced Prob|ems
yrohlem 0? Bomania I084|
Pladthenlnlnunol
lo, -

,-lo
, ~
,+ +|o, Xl
,
wa
ere Ji ~.
. .Jn arerealnunbersln the lnterval l )
5olut!on0?
B
3lnee lo~ ls a deereasln |unetlon ol x when 0 < < l and. s|nce
," i}!)
0 lnplles~
.
x l}1wehave

I
,

lo~+
,
lo, ~+, _ lo,+, =2 lo,~+
, =2
lo,
lt|ol|ows that
lo
-

-


,
-lo, ~+ ,-

+|o
-
_ ~, ,

lo~
.
lo~,
lo~
lo~,
, 2 + + ` ` + +
lo~, lo~
.
lo~, lo~
2n
j
ythe AM-GM !nequality
aalltlesholdiland only if
~, =~
.
=

=~ = }2
ProbIem 08 AIME I084|
0eternlne ~
.
-,
---o ll

Bo| t|oostoAdvaoc edProbe

SoIution 08
The elal n that the lvea systen olequatlons ls satlsed by 2
d

l

, , ,
an { r, ejurva ent to elarnu that

,
o
/ - i
+
/
|
-
i
-
-
/
= I
ls satlsed by / !,l 1.l1. and !
.

vultlplyln

to elear lraetlons we nd that lorallvaluesol t lorwhlehlt


l; de

ned | e , /= I , , ., and !) , I ) ls ejulvalent tothepolynonlal


equatron
P
'/ ) 0,
where
P'/)= '/ l ) '/ ) '/ . ) '/- !)

'/- ) '/- .) '/ !) ,

'/ l ) ' / .) '| !)

'| i ) '/ ) '/- !) o

'/ l ) '/- ) ' / .)


$laeedeP'/)=!, P'/ ) 0hasexaetlylourzeros / ! l1 3 aad!
l e ,
,
P,/) =(t 1) '/ l 1) ' / l1) '/ 1!)
Conparlnthe eoe1eleatsol/ ln the two expresslonsolP'/) ylelds
I --.+!-

+ ,

-o
=!+ I 1-l1+ !,
lronwhleh ltlollowsthat
Froblem 00 HaIkan I00?|
llnd allunetions
!
. * J sueh that
!'!')-!' ,
='!') ,-,
|orall E R
Solut!on 00
Let
!,) a $ettln .r 0 la the lvea eondltlonylelds
lorall ,E R
!'!' ,
a

-,.
$lneetheraneola+,eonslstsolallrealnunbers,!
nustbesur]eetlve
4. 5o|ut|oosto Advaoced Prob|ems
.
[hu, there exlsts |E sueh that
!, |) =
3ettln=|lnthelveneondltlonylelds
!'!',
!'|!' |)+!',
= '!,|

-,
arally E R lt |ollowsthat, lor all .,E ,
that ls,
'! '-

-, !'!')-!',

=!,!'!, ) )!')-!' ,) '


=
!,!') !'!'
+ ,'
!,!') )

+,=

-,.
B3
I )
ltlselearthat
!')=ls alunetlonsatlslylnthelveneondltlon
3uppose that
!') = Then thereexlsts sonenonzeroreal aunber

suehthat
!') -e $ettln
!' )+ !',) ln , I ) ylelds
, !'!' )-!',) ,!' )-!',) '

-!',) '
.
lcrall,E , and, settln=

ln the lven eondltlonylelds


!
(
!,-!,,
= (
!,c

+ ,= e
,.
larall ,E R
batethat '!' ,
=,

Itlollows that
ar
!',) ,

|orall,E J, alunetlonwhlehsatlses the lven eondltlon


Thereloretheonlylunetlonstosatl sy thelveneondltlonare
!') ~
ar
!')= lor E R
Problem ?0
The
nunbers I000, l00I ,

, .havebeen wrlttenonaboard
aehtl ne, onelsallowedtoerasetwonunbers,say,aand|. andreplaee
theabythenunhernl na, |)
|ter I sueh operatlons, one obtalns exaetly one nunber C on the
j
card
Frovethat e< I
`
B4
Solution ?0
Py synnetry, we nay assune a_ | Then
Wehave
nlna, |) =
l l

!ron wuleh lt !o|lows that the sun o!the reelpoealso!al| the nunbers
on the board ls nondeereasln l e the sun lsa nonovarlant )
At the belnnlnthlssunls
l l l l
S =
l
*
l;
*

+
.

where l } C lsthesunat theend Xotethat.lor l s/_
l
+
l
=
: : l
. / .
*
/ . /
>
.
~
l

Rearran|nterns|n5 ylelds
or c< l , asdeslred
Froblem ?I Hulgar!a I008|
Let a

a
berea|nunbersnota|l zero
Provethattheejuatlon
hasatnost onenonzerorea| root
Solut!on ?I
Xotleethat
!
, ,-)= ,ia,-ls eoneave Eenee
!,) vI a

- ,
*
a

-
4. 5o|ut|oos to Advaoced Prob|ems B5
ls
coneave
Sncc !' , ) exlststhereeanbeatnostonepolntontheeurve,=
!,)
wlthderlvatlve

3uppose therelsnore than one nonzeroroot


3lnce = lsa|soaroot, wehave threeeal roots-

< -

< -

Ap-
plylnthe Vean-Va|uetheorento
!,)onlntervals -

and -

,-


we can hnd two dlstlnetpolntson the eurve wlth derlvatlve .aeontra-
qctio
[herelore ourassunptlonlswronandthereeanbeatnostonenonzero
tea| rootlor theequatlon
!,)=n
Problem ?2 Trkey I008|
Let[a} bethesequeneeo!rea| nunbersdehnedby a,= t and
a
.. =:a
, ; a )
crn _ l
lcrhow nany dlstlnetva|uesol|dowehave a

,,, =:
5olution ?2, Alternative I
Let
!,)= :, l- ) 0bservethat

,)= [ l } ]

, l ) [ l}.} ]

, , l ) = l
and [,
!,,)=} = .lor al| -E . l )
LetA
= [ -E !. ,)= } . then
A.,
= [E !
.

,)= }
[E ! ), ) ) }= [E ! !, ) E A
We e|aln that lor al| n _ l , A

C l . | E A-.and
A
=.
-
+ 1 .
Icr n = 1 , we have
A

= [E ! ` !
,)= } [ l }
and theelalnshold
\
owsupposen _ 1 andA C ,. l

l E A
, and A
= .

` +l Then
E A. !,)E A, C , i -E , l
0 A
.

C . l
,
B6 4. 5o|ut|oos to Advaoced Prob|e

$inee],0) =], I ) = 0, we have ]


+
' , I ) =!ora|l n _ | so 1 E ^
+

.
Xow wehave
l~ ], ~) E ^


_
l~ ], ~

= a,
l~ ], ~) =I ,
+
_ l ~ ],~` ~ a
I +
_
2
aEA"
aErO, I )
1 +2,

^- ' I )
aEAn
aErO. l )
I + 2, 2

' + l
I )
2

-1 .
Thusthee|ainholdsbylnduetlon
lina||y. a, ,,, =i!andonlyi!J"", |) 0. sothereare2'
,,
+ I sueh
valueso!|
SoIut!on ?2, Alternat!ve 2
As in the previoussolution, observe that i!]~) E 0 I| then ~ E 0, I| ,
soi!a, ,,, a we nusthave | c 0. I[ .
Xow choosee E 0. z/2 sueh that sine = ,
0bserve that !or any E R,
sineea, =sin

e, i t!o|
j
ows that
There!ore
k
a,,,, = <= sin2
, ,,:
e = <= e =
2
,,,:
!or sone /E 7
Thusthevalueso!t whichivea, ,,, = are
/c 7, ivln2'
,,
+ l sueh values o!t.
4. 5o|ut|oos to Advaoced Prob|ems
w
B7
roblem ?3 IMO I00?short l!st]
a) lothereexist !unetions

l P and 9 R Rsuehthat
],q,~)) =~

and q, ],~) ` = ~`
!orall~ E R`
b) lothereexist!unetions] . R Rand9 R R sueh that
]'
q, ~) ) = x
.
and q,],~) ` =

!orall:7 E R`
5olut!on ?3
a) The eonditionsinplythat ], ~`` =],q,],~` ) ) =],~)|

whenee
~ E lI ,0.I ~` = ], ~` =],~),

],~)c (t ,
Thus, thereexist dierenta, 'E [-| , 0. : ,sueh that ],a) =],')
Put thena` =q, ],a) ) = q, ],'` ) = '` , aeontradietion
There!ore, thedesired!unetions]and9 donotexist
b) Let
{ |I xl i!
z
l
q~)= ~
- |
-
I xl !0 < ~ < l
r!~ =
Xote that 9 i seven and

'

whenever q,a) = q,') , thus, we


are a|lowedtodehne] as an even !unetion sueh that
],~)= y

. where issueh that q, +) = ~


We clainthatthe!unetions].9 deseribedabove satis!ytheeondi-
tionso!theproblen
tlselear !ron thedenitiono!] that]q ))=~

Xow let y ],~)


Then q,) =~ and
q, ],~) ` = q,

`
{
,

)
| - ,, ,
~

. - ,
=
. - ,

)
- | - ,,
=
- | - ,

q, )|

i! _ I
i!0 < y < 1
i! =
BB_____ 5out |oos to Advaoce d Prob|

Prohlcm 74 WcIchao Wu[


Le| 0
<
a, a
.
_ a

, 0
<
|, _ |
.
_ |, bereal nunberssueh that

_
a

_

.: .=
,
$uppose that there exlsts l _ k _ sueh that |, _ a, !or l < i <k and
|, a, !or |>k.
-
Prove that
SolutIon 74, AltcrnatIvc I
We dehne twonewsequenees lor i l 2 !, .et
Then
or
/
d |
'
|,a
a- aan _ " " _
a.
There!ore
a =a - a
.
'
- - a' > |
'
+ |
'
-
. . +
|
'

,
ApplylntheAM-GM IncqualIty y!elds
| |
| a


I I I
i

|
'
|
'
.
|
'

<
|
i
- |
.
t

- |

a, a
.
a

_ a
!ronwhleh thedeslredresu|t!o|lows
SolutIon 74, AltcrnatIvc 2
Wedehnetwo newsequenees lor i ~ I 2 . , .et
a

a and |

|,- a - a, > c.
Then
| - |_+ . . . + | ak
Xotethat, !or c, x- ) - e) _ c.
x x + C
. x >
and c> c
Y - e
- -

i )
4. 5o|ut|oostoAdvaoced Prob|ems B9
3ett|nx= a, , Y " |, andc a a, , theabovelnequa|ltylnp| lesthat
, }| <
a|, !or i " l2, . , Thus,
2)
Lsln l) andtheAM-GM IncqualIty yle|ds

3)
t lse|ear thatthedeslredresult !ol|ows!ron 2) and,3)
Prohlcm 7
0lven elht non-zerorea| nunbers a, ,a
.
,a, . prove that at |east one
c!the !o| |owln slx nunbers. a, a

+ a
.
a, . a, a, + a
.
a, a, a- + a
.
a, ,
a

a, + a, a, , a

a-- a,a, , a,a-+ a,a, isnon-neatlve


SolutIon 7 Moscow OlymIad I078[
llrst welntrodueesonebasle know|edeo!veetor operatlons
Let u " [ah| andv ~ m. n be two veetors
Iehnethelr dclrcduc|u v am - hn

t lseasyto eheek that
l) v v m
z
-n
z
" v
z
, thatls.thedotprodueto!veetorwlthltsel!
lsthesquareo!thenanltudeo!v and v v _ 0 wlthequallty l!
andon|yl!v = [0, OJ ;
(ii) u v " v u,
(iii ) u v + w) = u v + u w where lsaveetor,
lv) ) v = eu v , whereC lsasea|ar
When veetorsu and v are plaeed tall-by-tall at the orlln O |et A and
be the tlpso!u andv. respeetlve|y Then = v u

Let /AOE

^
^pp|ylnthe law of cosIncs to trlan|eAOEyle|ds
v u
z
" AE

OA

OE

- 2OA OE cos ^
u
z
- v
z
2 u v eos ^
@@@@ o| u t|oostoA dv aocedProb|e

lt !ol lows that


v u
v u) = u u +V v 2 u v eos ^,
or
u v
eos ^ =
u v

Consequent|y, i!_ 0 .U y _ 0
Conslder veetors Vl = al , a

| , v

= a+ , a,| , v+ = a, , a,| . and V4


a- .a, |
Xotethatthenunbersa, a+ +a
.
a, a, a, +a
.
a, , a, a- +a
.
a, ,a+a, +a,a ,
a+ a-+a,a, . a

a-+a,a, are all the dot produetso!dlstlnet veetors N


andVj .
1
$lnee there are !our veetors, when p|aeed tall-by-tall at the orlln. at
le

t two o!then !orn a non-obtuse anle, whleh ln turn lnplles tae


desrredresult
Prohlcm 70 IMO I000 short list|
Let a, | and c be posltlverealnunberssueh thata'e= I
Provethat
a' 'c ca
a +|+a|
+
' +e +'c

c+a
,
+ca

'

SoIution 70
We have
beeause
a
+
- '`) , a

'

) _ ,
w|thequa|ltyl !and only l!a= '
,
Eenee
Llkewise,
a' a'
a+|+a|
>
a '
,a+')+a'
I
=
a|,a+|)+ 1
a'c

a|,a+'+ C)
e

a + ' + e
|c a

|+ c +'c a+ '+e
4. 5o|ut|oosto Advaoced Prob|ems
.
an
ca
'
-- <
.
c + a
,
+ ca
~ a+'+c
3dlnthelast three lnejualltles leads tothe deslredresu|t
ualltyholdsl!andonlyl!a '=c= I
Commcnt. Please eonpare the solutlon to thls problen wlth the
secondsolutlono!problen I3lnthlsehapter
Prohlcm 77 Czcch-Slovak match I007|
Plndall!unetlons] . l l sueh that theequal|ty
]'], ~)+) =],~

)+1],~)
aelds!oral| palrso!rea| nunbers J
Solution 77
Clearly. ],~)= ~
satlses the!unetlonal equatlon
New assune that there lsanonzerovalueasueh that ],a` = a

Let=
~

|nthe!unetlona|equatlontohndthat
]

],)
2
+~

=]
],~)
2
+

,
2]'~)
,
]'~) )
0~ 2],~) ~
- ]'~) ) Thus,!or eaeh~. elther], ~) =or], ~) =~
.
In both eases. ],0) " 0
3ettln~ a, lt!ol lows!ron abovethat elther],a) =or],a) or
),a)" a
z

The latter ls!alse, so ],a)


bow, let~ andthen ~ " a .nt he!unetlona| equatlon tohndthat
],) = ],) . ],)

],a

)
aadso
],) ]' ) =]'a

+)
that
ls, the !unetlon ls perlodlewlthnonzeroperloda
Let a
.
ln the or|lna| !unetlonalequatlon toobtaln
],],~)) ],],~)+a
.
],~

a
.
,+1a

],~)

],~

)+1a

],~)
ewever, puttln ln the!unetlonalequatlonlves ],],~)) ],~

)
|erall~
92
4. 5o|ut|oosto Advaoced Prob|e

Thus, 1a
]~) =0 !orall x. $lneea lsnonzero, ], x) =0 !or a|| !
T
j
ere!ore, elther],~)=x2 or]t) =a
Prohlcm ?8 Ktan||
$olve thesysten o!equati oas
3~-
~ + = 3
~+
X +3
- 0
~
+
-
SolutIon ?8, AltcrnatIvc I
\lu|tlp|ylnthe seeondequatlonbyi andaddlnit tothe hrst equatlon
ylelds
or
+

+
3~ - ) - ~ + 3)i
~
3
~
+

-
+

+
3~ - i) i~ - i)
~ t
3
~ +

~
+

Let =~+i Then


~ i
:
=
~ +
Thusthelast equatlonbeeones
or
Eenee
3 - i

+ = 3

- 3
+ 3 i) =o.
=
3+v-3+ 1i
=
3+, I +
2i )
2
2
that ls. ~ = 2. I ) or ~,
= I

Solution ?8, Altcrnativc 2
Vu|tlplylntherstequatlonby, theseeondby~, andaddlnupyle|ds
2~+
3~- ) ~+3)~
=3
_
+

or2~- I =3. It !ollows that / 0 and


3+I
, =
_o|ut|oos to Advaoced Prob|ems 93
,ub
stltutlnthlsiatotheseeond equatloao!thelvea systenlves

3+ I


3
v+
I 3 0 - + - -

.
2 2
cr
1' - 3 I =a
It !ollows that

I and that the solutloastothesysten are 2, I)and


I , -I)
rohlcm ?0 China I00|
Hrlat and Vr Ta!play aanewithapo|ynonla|o!dereeatleast1.
They hll|nreal nunberstoenptyspaeesln turn
I!theresultlnpolynonlalhasnorealroot \ir lat wlns. otherwlse,Vr
Ta! wlns
I!Hr lat oesrst.whohasa wlnnlnstratey!
SolutIon ?0
HrTa! hasawlnnlnstratey.
Wesayablankspaee |s cdd, ctcn) l!lt ls the eocdelento!anodd even)
pcwero!J
P|rstVr Ta|wllllllnarb|traryrealnunberslntooneo!therenalnln
evea spaees, l!thereareany
S|nee there are only n I even spaees there wi | l be at least one odd
spaee le!t a|ter 2r 3 p|ays. thatls, thelvenpolynonialbeeones
where and 2t I are dlstlnet posl tlve lnteers aad ,~) ls a xed
clynonlal
\c elalnthattherelsarealaunbera suehthat
wlllalwayshavearealrootreardlesso!the eoedelent o!~

:-;
,
Then Vr Ta!ean slnply hll ln ln !ronto!~` and wln the ane.

94 gg4 o| utoostoA dv aocdProb|e

Xow weprove our elaln Let |be theeoe|eot olX


2t -1
|op(x
) . Xote
that
I
2
2t-1 P
( 2) + j'l )
=

2
2
'@
1
q(2) +
2
5 -
2t+
1 a + |
;+ [q(- l ) + (
-
1 ) 5a - |
=

22t
1_1
q(2) + q( + a[
2
8 -2t
+1 + -I )
|
$loee

2t - I , 2
8 -2t +1
+

-1
)
8

O.
Thus
a =
I
2
q( 2) + q( -I )
2
5 2t+1
+ (

I )

!s wel| dehoed sueh thata ls lodependeot o!| aod


I
22t- 1 P
(
2
) + j
l ) =O.
lt !ollowsthat eltherj-I ) p(2) = 0orj'-I) aodp(2) have dlereot
slos. whleh lnp|les that there .sa rea| .oot o!j
) lo betweeo -I and
2.
loelt|er ease p(x) hasa rea| root reard|esso!theeoee|eoto!
x
2t-1
,
as elalned
0ur proo! ls thus eonplete
Prohlcm 80 IMO I00? short lIst|
l|od a|l posltlve loteers k !or whleh the!ollowlostateneot |s true |!
F(x) lsapo|yoon|al wlthloteer eoee|eotssatls!y|otheoodltloo
O $ F(e) $ k !or e = O, l , , k + l ,
theo F(O) = F( l ) = . = F( k + I )
SolutIon 80
Thestateneotlstrue.!aodooly| !k 4.
We startbyprovlothatl tdoesho| d!or eae| k 4.
oos|der aoy polyoonlal F( x) wlth |oteer eoe1ie!eots sat!s!ylo the
aequallty0$ F( e) $ k !or eae| e E {O. l k+ i }
Xote hrst that F( k + I)= F(O) , s!oee F(k + I) - F(O) |s a nultlp|e
o!
k+ I notexeeedlok lo aosoluteva|ue
Eenee
F(x) - F(O) = x(x - k- l ) G(x) .
4. 5o|ut|oost oAdvaoced Prob|ems
~
w|ere G(x) ls apolyoonla| wlth loteer eoeeleots
Censequeot|y.
k I F( c) - F(O) I = e(k + I - e) I G( e) 1
ereaeh e E { l2 . . . . . k} .
95
I )
[|eequa|ltye k+ 1 - e) > k ho|ds!oreaeh e E { 2, 3 . , k l } as | t ls
eulvaleot to e - l )
(
k - e) > O.
Netethattheset {2. 3 .k l}lsootenptyl!k 3,aod!or aoye lo
t|lsset I) lnplles that I G(e) 1 <
1
.
3ineeG(e) lsao lnteer. G(c) = O.
[|us
F(x) - F(O) = x( x - 2) ( x ) (x - k+ l ) (x k- l ) H(x) .
w|ere H(x) ls po|yoonla|wlt|loteereoeeleots
Teeonp|ete t|e proo!o! our e|al n |t renalos to show t|at P' l )
H(k) =O.
Netethat !or e = I aode= k ( 2) lnp||esthat
k I F(e) - F(O) I = k- 2) !

k I H( c) l
ler k 4 , k - 2) ! > I
eoeeH( e) = O.
( 2)
We establlshed thatthestateneot . othequestloo holds!or aoy k 4.
@ut the proo!a|so provldes lo!ornatloo !or the sna|ler va|ues o! k as
well
Here exaet|y. l!F( x) satlshes t|e lveo eoodltloot|eo 0 aod k + I are
reots o!F(x) aod F(O) !or aoy k I . aod l!k 3 theo 2 nust also be
sroot o!F( x) - F(O).
Taklo t|l s| otoaeeouot, |t lsoot|ardtoodthe !o|lowlo eouoterex-
saples.
F( x) = x(2 - x) !or k = I .
F(x) = x( 3 - x) !or k= 2.
F(x) = x( 4 - x) ( x - 2 )
2
!ork=3
l
96 4. 5o|ut|oosto Advaoced Prob|e

Prohlcm 8I Korcan Mathcmatics Comctition 200I|


Thellbooaeelsequeoee| lslvcoby
Prove that
!oral|n _ 2
Solution 8I
Xotethat
whence
!oralln _ 2
|+.
+|. _
2|

|.
=
.
'

I)
$ettloa= J|z
,|~
|.+.
, aod c -
|..
l othealebraleldeotlty
a

+ |

+ c

- Ja|c a+|+c) a
.
+|
.
+c
.
- a| |c- ca)
lves
2|; +.
- |;.
- '|.+. |.|..
0
Applyln I ) twlce lves
|z+zz. |
= Jz |.=.
)
|..
-
|,

|; ,J|z=;
- ; )
|,;
=
;|;=,
|,;
~ ' '

=F,
.
- |_ ~ -I
The deslred resu|t!ollows !ron
_5o|ut|oosto Advaoced Prob|ems 97
rohlcm 82 Bomania I008|
llod a|| !uoetloosu P P !or whle| there exlstsastrletlynoootoole
unetloo ] P Psueh that
!era|| x, E R
Solution 82
]x+) ]x)u,)+]
Theso|utloosare u, x) =a a E P
+

Tosee that these wor|, take], x) x!ora I


l!a= I . take ]x =a - I . theo
]x-) =a
+
' I =a - I a'+a' I =]x)u
) +])
!er al| x, E R
Sowsupposeu P P, ] P P are !uoetloos!or whleh ] lsstrletly
moootooleaod ]x+ =] x) u)+]) !or a|lx, E R
we nust show that u lso!the !orn ux) =a 'or sone a E P
+
llrst,

lettlo y =0,weobtalo ]x) ]x)u 0)+ ]0) !oral|xE R


Thus, u0) = I wou|d .np|y ]x) ]0) } I u 0) ) !or a|l x, whleh
wou|d eootradlet the!aetthat ] lsstrletly noootoole, sowenusthave
o0) I aod ],0)=0
T|eo ]x) = 0!orall J = 0
Sext ,wehave
],x)u)+]) ]x+) =]x +] u x) ,
er
]x) u)- I ) ])ux)- I )
!or al|x , E R Thatls,
!or all x = 0
ux)- 1
]x
u) - I
] )
It!o|lowsthatthereexlstsCE P suehthat
|era|lx = 0
u x)- i
=C
]x)
Thus,ux) l +C]x) !or x= 0 sloeeu 0) I ]0) 0, th|sequatloo
slsoholds!or x=0
9B 4. 5o|ut|oosto Advaoced Prob|e

llC =0, thenu(x) = I lor al| x, andwe are done


0therwlse. observe
lor allx, y E P
u(x + y) = l
~
Cf(x + y)
I +Cf(x) u(y) + Cf(y)
= u(y) + Cf(i)u(y)
= u(x)u(y)
Thusu(nx) u(x)
n
loral| n E Z, x E l
$loee u(x) = I +Cf(x) lor all x, u ls strletly noootonl e. aod u( -x) ~
l /u(x) !or all x, so u(x) > lor all x u(O) I
Let a u( l ) > OJ thenu(n) =a
-
lor all n E N, aod
u(p/q) = o
/
) ) ' a'
lorallp E Z, q E N, sou(x) a
X
lorall x E Q
$l oeeu lsnonotonlcand theratlooalsaredenselo P,wehave u(x) =a
lor allx E P
Thusall solutlonsareo|the lornu(x) = a, aE P

Prohlcm 88 China I080|


Let :, ,:

,. , :

be conplexounbers such that


Provethattheteexlstsasubset 5 ol{:; ,:

, . . z
n
} suehthat
Solution 88, Altcrnativc I
Let l, ,l , and l

bethree rays!ron orlln that lorn anglesol00 ,l


aod300, respectlvely, wlththeposltlve x-axl s
lori= I , 2, 3,let'i denotetherelenbetweeol,andl,., herel, =l, ` ,
loeludlnthe ray l, Then
Bythe Pleonhele Prlnclple, at leastene o the above suns lsnot less
than I {3.
5o|ut|oos to Advaoced Prob|ems 99
3ay lt's '3 otherwlse. we apply a rotatloo. whlch does oot eect the
anltudeolaconplexounber) Let: =

+i
v

Theo !or z E '3 ,


x - '
=
:
/2.
Censequeotly.
@ deslred
Solution 88, AItcrnativc 2
Weproveastroogerstateneot there .ssubset5 o! |:; ,:, , . , :

sueh
that
lor l _ k_ n, let :

+i
v
Theo
I =

:,
+
, ~
+

> , : + v
, ) +, '

+
v

)+

-,
~
+ v `
_

+
_

+ _' v

~ _ v


Xk O Xk <O YkO Yk <O
@ythe Plgeoohole Prloelple. at least ooe olthe above suns ls oot less
thao I /1. By synnetry, we nay assune that
Coosequeotly,
Commcnt. Lsl og advaoeed nathenatles, the lower bouod ean be
lurther . nproved to I /
Prohlcm 84 Czcch-Slovak Match I008|
polynonlalP(x) oldegreen _ 5 wlthloteger eoeeleotsaodn dlstloet
loteger rootslslven
lloda|ll oteger rootsolP(P(x) ) glveothatlsa rootolP(x) .
l
00

Solution 84
Jhe roots ol P,)are clearly i nteger roots ol P,P,r) ) . we claim there
are no other integer roots
We prove our claim by contradiction Suppose, on the contrarv that
PP, /) ) " 0lor some integer / such that P/) ; O.
w ,
Let
P-) a , ) ' z

,)
where a. r, ,z , , , are integers,
, =0 rz ' > ' >

5 ' , '

SinceP,/)/0, we must have /~ , ,_ 1 lor all i


Sincether,are all disti nct, at most twool /

/- / , equal
1 , so
a,/ z

/- ,
-.' a / r

/ / , .
and P, /)
_ . //~ , '

Alsonote that P /) = r,

lor some i, , so , P, /)
,

Now we consider the lol lowing two cases.


I
/' ` r,

Jhen

P, /) , _ .' /,/ ,) , _ .' /' > j r,j , a contradic-


tion
2. / < that is, 1 / ' , ' 1 .
Let a. b, C bereal numbers,
a b. For E a b] , the lunction
!) )
reaches its minimum value at an endpoint aor b, or at
both endpoints

Jhus
t lollows that
which impl ies that , _ . Since n _ . this |sonly possible il
P-) = +.) l ) l ) - .)
ut

then | ti simpossible t ohave / / and , /, j r, , a contr


diction
4. 5o|ut|oosto Advaoced Prob|ems 0
.
|husourassumptionwasincorrect, and theinteger rootsolP,P, , )are
ex
actly al | the integerrootsolP,)
rohlcm 8 Hclarus 1000|
|worealsequences~, .~
.
, andy, . qz . ,aredennedinthelol lowi ng
wa)
nd
~,
l ~, ,t+~ ,
q,
q,+i

1
+ ,iy
lorall n _ I Irove that .< ~,), < J lor all n > I
Solution 8, Altcrnativc I
[et z, lJq, and note that the recursion lor q, is equivalent to
Zn+l
= :, + ,:
Also note that :z = , = ~l . since the ~,s and :,s satisly the same
recursion. this means that :, =~,. , lorall n > I
Thus,
Note that
D -,
~,,
-

`
^D
!
-i
_J ~
=,
> Xn-l
Thusx
- > .~,[ and ,q, > . which is the 'ower boundolthe desired
luequality
oince ~,s are increasing lor n > I , we have
which impliesthat
.-
-
> _lX,_l
'
husJ~,
, > ~, . whichleadstotheupperboundolthedesiredinequal-
ity
Solution 8, Altcrnativc 2
oetting ~, =cotP, lor 0 < P, < yields
Xn+l
=cotP, ,icot
z
P, = cotP, csc P, " cot
0
.
,
,
M
02 4. 5o|ut|oosto Advaoced Prob|e
s
-
Sincc(h = !J , we have ingeneral ^, = _ Similar calculationshows
that
It lollows that
2 tan^,
), = tau, 2^, =

1 tan ^,
2
:
, ), =
l tan^,

Since tan^, = 0 tan^, is positve and r,), > 2
And since lor n > 1 we have ^, < !J wc also havc
so that :,), < !
Commcnt. From the closco lornslorr, and), inthesccond solution,
wecan see the relationship
used in thenrst sol ution
Prohlcm 80 China I00b|
l
)
, =
:
For apolynomial P, ) dennethe4icrcnccolP ) ontheinterval a'|
' a' ,a' ,a' as P''; I
,a)
Irove thal i ti spossible to dissect theinterva| J | intoann| te number
olintervals and color them red and bluealternatelysuch that. lor every
quadratic polynomial P' the total diUerence ol P' on red interval
is equal to that olP, ) on blue intervals
What about cubic polynomials!
Solution 80
For an interval i. let /,P denoteIhe diUerenceolpolynomialP on i
For a positivereal aumber c and a set 5 R, let 5+C denote the set
obtained by shilting 5 in the posi tivedirection by C
We prove amore general result
Lcmma
Let l be a positive real number, and let k be a positive integer It is
always possible to dissect interval ] =
J. 2'|| into a nnite number o
intervals and color them red and blue alternatively such that. lor every
polynomial P)wth degPk, the totaldherenceoIP,)on thered
intervals isequal to that on the blue |ntervals

5o | ut oostoAd v aoedP obes


mmmmmmmmmmm

]rcc{
ye inductonk
Ior k = I. we can ust use |ntervals 0. lj and l. 2lj I t i seasy to see
|hat alinear or constant polynomial has the same diUerenceon thetwo
nter\ls
Suppose that the statement is true lor k = n. where n is a positive
nteger that is. there exists a set F, olred disoint intervals and a set
b-
olblue disoint intervals such that P, n E, = O. P, . E, l, and.
forany polynomialsP') with degP n. the totaldiUerences olP on
] is equal to thatolPon E,
bowconsider polynomial {:) wth degf + I. ehne
), : = f(x + 2
,
l and |,: = {,:
g
')
|hen deg
h
n. By the induction hypothesis.
or
_
6bh =
_
6rh,
bEBn rERn
_
6bf +
_ 6rg

_
6r f
+
_
6bg
bEBn rERn rERn rEBn
Itlollows that
where
_
6b
{
_
6r
f,
bEB+l rER+l
F = P, U ,E, +2
,
l ,
and E, = E, U ,F, +2
,
l
IlP_ and E both contain the number 2
,
(, that aumber may be
removed lrom one olthem )
I t sclear that E, and F lorm a dissecion ol I, and. l

r a

y
olynomal { with deg{ n + l , the total dtUerence ol { on E, 1
qual tothat ol{ on P
heonly possibletroublelelt isthatthe colors in E .F
.
might not
bealternati ng whichcanhappenattheendoltheI, andthebeginning
ofI, +
2
,
(


but note that ilintervals i, = c, . ', j and iz = ', . c j are in the same
color. then

6i) { +
6i2 f = 6i3 f.
04 4. 5o|ut|oos to Advaoced Prob|e

1hus inB.P_ ,wecansimplyputconsecutivesamecolor intervals


intoonebigger interval olthesamecolor
1hus, thereexistsadissection
such that, lor every polynomial !' )with deg
!
_ +1 .
Jhis completes the induction and theproololthelemma 0
Setting nrst C = and then C = in the |emma. it is clear that the
answer to each olthe given questions is yes "
Prohlcm 87 USSB J000|
Oiven cubic equation

-_
+_+ = O.
hr Fatand Mr 1al areplaying thelollowinggame
In onemove, Mr Fatchoosesa real number ano Mr 1al puts it in one
oltheempty spaces
Alter three moves the game is over
Mr Fat wins the game il the Gnal equation has three distinct integer
roots
Who hasa winning strategy:
Solution 87
Mr Fat has a winnlngstrategy
Let the polynomial be

+ a -|+C. hIr Fat can pick 0 nrst We


consider thelollowing cases
a) Mr 1al chooses a= 0. yielding thepolynomial equation

+ |+c = O.
Mr Fat then picks the number ' noj) . where . and are
threeposi tive i ntegers such that

+ o= j

Il Mr 1alchooses |= ,oj)
then Mr Fat will choose c = O.
1hegi ven polynomial becomes
' woj) ,+ oj)
4. 5o|ut|oosto Advaoced Prob|ems
IlMr 1alchooses c = -,oj) . then Mr Fat will choose
1hegiven polynomial becomes
b) Mr 1al chooses |=O. yielding theequation

+ a+C ~ O.
Mr Fat then picksthenumber

'-i )

,n + m+ |

where : is an integer greater than I


IlMr 1al chooses
a=
' -i )
, + + i )

then Mr Fat can choose


1hepolynomial becomes
' j) ,+, + i )j| ,+'n+ :j|
where
Il Mr 1al chooses
c =
'm+ i )

'
-+ i )

.
then Mr Fat can choose
1hepolynomial becomes
,-)) , -i ) )' , ,+i )'
where
)= ++ i

05
\06 4. 5o|utloosto Advaoced Prob|e

c) Mr 1al chooses C o.
1hen the problem reduces toproblemaol the previous chapter.
Mr. Fat needs only to plcktwo lntegers a and such that
,- : , , - : , = a
and a += -:
1hepolynomlal becomeselther -,-- : , ,-- a) or -,-~ : , ,-- ,
Our proollscomplete.
Below isan exampleofwhat Mr Fat and Mr 1al cou|ddo.
F 1 F 1 F Roots
a a -:aa -a aa
" "
L -:-+ z:
"
+ : -s z: +
- z: +
s + +
"
L -+ -: z: z
"
C z a -: -: a

:
" " "
-: a t z
Prohlcm 88 Bomania I000|
Let> zbeanlnteger and let): s-sbea lunctlon such thatlor
any regular n-gon z,z

z
-

Irove that )ls the zero lunctlon


Solution 88
We identllyswith the complex planeand let ( =

1hen the conditlon ls that lor any -E l and any posltlvereal


-
_), - + , , = a
j=l
Inpartlcular, lor each ol/=: weobtaln
-
_) -- (
k
+(
j
) =a
j=l
_5o|ut|oosto Advaoced Prob|ems
Summngover / wehave
- -
__), -- , t - (
m
)(
k
) =a
m
=l k=l
\07
IorJ =the innersumls),-, lor other J thelnner sum agaln runs
over aregularpolygon, hencels .
|hus ), -, =alor all -E l
Prohlcm 80 IMO I00?short list|
Let ;be aprlme number and let ),-,be a polynomlal ol degree d with
inIeger coemcients suchthat.
l) ),a,=a ), t , =1;
,ii) lor every posltive integer the remalnder upon dlvlslon ol ),,
by;seither aor t
Prove thatd _ ;- :
So|ution 80, Altcrnativc I
Ior the sake ol the contradlction, assume that d _ ;- z
|hen by Lagrangc's intcrpolation formula thepolynomlal ),-,is
determinedby ltsva|uesata1, ,;- z.thatis,


- - / + : , ,- - / - t ,

,- - ; - z,
),-, =

),/,
/
. l
.
(- I)

,/ - ; + z,
k=O

X
,-- /+1) ,-- /- t, ,-- ;+z,

)/,
k! ( -1 )
p
-
k
,; - / - z,

k=O
oetting -=;- :gives

,; - t , ,; ~ z,

; - /,
)
'
; - :, =

), /,
,~(
k=O
Itlollows that
p
-2
. ~i )"
_),/,
mod;,
k=O
, ), =),a, +), t , - +),;- 1) ~ a ,mod ;,

( 1 )
0B 4. 5o|ut|oos to Advaoced Prob|es
"
On theotherhand, ii)implies that S( f) mod ) , wheredenotes
the number olthose
k
E 0. 1, . . - l lor which f( k) I ,mod j
.
But i ) implies that I p - 1 .
So S( f) =O mod ;, which contradicts ( 1 ) .
1hus our original assumption was wrong. and our prooliscomplete
Solution 80, Altcrnativc 2
Again, weapproach theproblem indirectly.
Assume thatd p - 2, and let
f(x) =ap_2x
p-2
+
. .
'
+
alX +ao.
1hen
p-l p-l p-2 p-2 p-l p-2
S( f) = f( k) = ai ki =ai ki =ai Si '
k=O
)~j
whereSi = k
i
.
k=o
k=O i=O i=O k=O i=O
We claimthatSi O mod p) lor alli O. I , - 2.
Weusestrong inductiononi to proveourclaim
1hestatementis true lor i=O asSo =
Now suppose that So = SI Si-l = O mod) lor some 1 i
" 2. Note that
p-l p-l
O
_
p
HI
=
L ki+1 - L ki+1 L[ (k
+
l )
i
+1
- k
i
+1 ]
k=! k=O k=O

kj ,i+ 1 ) Si +
i+ 1

S
j
k=O )'=0 J
)'=0 J
,i+ l )S mod p)
SinceO < |+ 1 < p, it lollows that 5, O mod ) This compl etes the
inductionand the prool oltheclaim 1herelore,
p-2
S(f) = aiSi O mod)
i=O
1herest is thesameasin the hrstsolution
4. 5o|ut|oosto Advaoced Prob|ems
w
rohlcm 00
[et n be agiven positive integer
I
Consider the sequence ao. al . . , an with ao = _ and
lor k = 1, 2,
. .
. , n.
Prove that
1
1 - < an < 1.
n
Solution 00, Altcrnativc I
We prove a stronger statement. For k =1. 2 .
. . . . n,
n + l n
< ak
< k '
2n - k + 2 2n - `
We use induction to prove both .nequalities
Wehrst prove the upper bound |or k =1 , it iseasy to check that
ouppose that
1 1 2n + I n
al =

+
4n
=
< 2n - I

n
a
k
<
2n - k '
for somepositive integer k < n. 1hen
<
<
ak
(n
+
ak )
n
2n k
n +
2n k
n( 2n -
k
+ 1 )
( 2n - k) 2
n
2n - k - l '
(2n - k + 1 ) (2n - k - 1 ) (2n - k) 2 - 1 < (2n - k)
2

Thusour inductionstep is complete In particular , |or k

n - 1,
n

an =akl <
2n (n l ) - 1

09
( 1 )
J
0 4. 5o|ut|oos to Advaoced Prob|e

as deslred
Xowweprove the upper bound lor / lt lseasy to eheek that
$upposethat
2o- o- l
c,

>
2o - I
o - I
c `
2o~ /-2
lorsoneposltlve.nteger/< o Theo
c o - I ,o + l )
:
c+, c > -
2o /-2 o,2o /+ 2)
:
I tlollows that
o - I
c+,
2o~ /+i
o + I + I)
:
-
,2o /+ ,2o /-2) n,2o /+ 2)
:
o-I o- 2o /-2
2o- /-2)
:
o

2o~ /
o- ,

l
>
0
2o- /-2)
:
o 2o /- I

Thlseonpletcthelnduetlonstep lnpartleular,lor/ weobtala


o-

o-

>

c- =c+1 >
2o ,o I) - l o-2 o-2
asdeslred
Solution 00, Altcrnativc 2
Rewrltlng thcglven eondltlonas
o
c
-
; ,o - c, c
-
, o c
-
;
ylelds
lor/I, 2, ,o

l t lselcarthatcs are | nereaslng
Thus
]
c- > c-
-
; >

>c, _
(2)
_5o|ut|oosto Advaoced Prob|ems
[hus , 2) lnplles that
1
- - < -
c; c o
ler/= I , 2, ,o
|cleseoplogsunnatlonglves
er

>
I

2 I


c- c,
that ls, c, < whleh glves thedeslredupper bouod
Slnee c- < aod, slneecsarel oereaslog.
k=l , 2, ,o

c, < c
2
Then ,2) lnplles
:
- >

c
-
; c o - c
-
, o - I
ler /

I, 2, , o
Teleseoplog sun glves
o
er
thatls,
-
> -
c, a- o-I
o n - 2
" < * " "
c, c, o-I o- 1 '
o- I
a >

-> - -
,
o - 2 o + 2 o
whlehls thc deslred lower bound
Prohlcm 0I IMO I000short list|

_ c, < I lor
Letc, , c
:
,

, c, beooooegatlvereal nunbers. oot all zero


,a) Prove that :
-
c,x
,
-
' c,., x c, " 0 has preelselyooe
posltlverealrootu
b) let ^ ,,
c
aod E
-,1c,

Prove that ^` _ u
II2 4. 5o|ut|oostoAdvaoced Prob|c

Solution 0I
a) Cons|dcr thc lunetlon
Xotcthat) dcercacslron to0 x lnercascslron 0 toC,
EcnecthcrclsaunlqucrcalnunbcrFsuehthat)( F) 1 , thatls
thcrccxlstsaunlquc posltlvcrcal root Folthc glvcn polynonlal
b) Lct Cj aj/A.
Thcn
CjS arcnon-ncgatlvc and L c
J
= I
$lnec- lnx l s aeonvcxlunet|ononthc|ntcrval(0, 0) , byJensens
incqua|ity,
t Cj
-
ln
,j ) - l
n
(t
Cj ) = - |n, ) F) ) = O.
J=1 J =
1
l t lollows |hat
or
n
: Cj ( lnA +lnF) _ 0
j=1
n n
:c
J
|n A :jCj ln F.
j=1 j=1
$ubstltutlngCj = aj /A, wcobtaln thc desircd lncquallty
Commcnt: Plcasceonparc |hcsolutionola)wlththatolthcproblcn
I 5ln thc last ehaptcr
Prohlcm 02
Provc that thcrc cxlsts apolynonlal P( x, y) w|th rcal eocelcnts suea
thatP(x, y) 0 lor all rcal nunbcrs x and y, whleheannotbcwr|ttcn
as thc sun olsquarcs olpolynon|alsw|thrcal eocelcnts
Solution 92
Wcelaln that
l
P( x, y) = (x2 + y2 - l )x2y2 + 27
lsapolynonlalsatlslylng thc glvcneondltlons
l|rstwcprovcthatP( x, y) 0lorallreal aunbcrsx andy.
4. 5o|ut|oos to Advaoced Prob|ems 3
llx2 + y2 - I 0, thcn lt ls elcar that P(x, y)
>
0; |l x2 + y2 - I < 0,
thcn apply|ng thc AM-GM incquality glvcs
or
( 1 2 2
) 2 2
<
. x2 - y2 + x2 + y2

3
_
- x - y x y

3
27
'
1
(x2 + y2 - l
)
x2y2 _
27
'
Itlollows that P( x, y) O.
wcarclclttoprovcthat P(x, y) eannotbcwrlttcnasthcsunolsquarcs
olpolynonlals wlth rcaleocelcnts
lor thcsakcoleontradletlon,assuncthat

P(x, y) = : Qi(X,
y)2 .
i=1
$lnecdcgP = , dcgQi 3.
[hus
Qi (X
,
y) AiX3 + BiX2y + Ci xy2 + Diy3
+E,x2 + Fi xy + Giy2 + Hi x + 1iy +
Ji
Conparlng thc eocelcnts. ln P( x, y) and L
=1
Qi(X, y) 2, ol tcrns x6
andy6 glvcs
i=1 i=
1
or , = D, = 0 lor alli
Thcn, eonparlngthoscol X4 andy4 g|vcs
orEi = Gi = 0 lor all i
n n
:E; :GT = 0,
i=
1
i=1
Ncxt, eonparlngthoscolx2 and y2 glvcs
crH
i
^ Ii = 0 lor alli
Thus,
n n
:H; = :1; = 0,
i=1 i=
1
,
II4 4. 5o|ut|oostoAdvaoced Prob|e
9
Put, nally, conparlng thecoemelentsolthe tern , wehave

_
/:
=

:
|=i
whlchl slnposslblelor real nunbers /,

Thus ourassunptlon lswrong, andour proollsconplete


Proh|cm 03 IMO I000 short list|
lor each posltlve lnteger , show that there exlsts a posltlve .nteger /
such that
/=), , + :

,, ,

- :
lorsonepolynonlals),wlthintegercoeclents, and ndthesnallest
such /asalunetlon ol
Solution 03
llrstweshow thatsucha/exlsts
Xote that - dlvldes :-
Then lor sone polynonlal .,wlth
lnteger coeclents, wehave
,l-: .,:= i-

. , t

:
or
.= ,.,+,:


Ralslngboth sldes to the .` power , weobtaln
where,lsapolynonlal wlth lntegercoeclents
Thlsshows that a /satlslylng the eondltlon olthe problen exlsts Let
/,bethenlnlnunsuch /
Let .= .

q, where ls aposltlvelnteger andq lsan oddl nteger


Weclalnthat /,= .
llrst we prove that .dlvldes /, Let | = . Xote that


,+ , where
, =-

- -
-


-
:
Therootsol iare
.
-
cos : su

. .
.
t |

" . . , | ,
@5o|ut|oos to Advaoced Prob|ems II5
that ls,
u,:

t
,- ., ,-~ =

, - .


Let),and,, be polynonlalswlthlntegercoeclents such that
/,
ltlollowsthat
),,+ :

,, , ,

), , : ,
,, , ,
:
Slnce l sposltlve,t l seven $o
, :
Slnce),.

, ),.


),.

lsasymmctric polynomial ln., .

wlthlntegercoeclents, ltcanbeexpressedasapolynonlalwlthlnteger
coeclents ln the clcmcntary symmctric functions ln ., .

snd therelore
lsan |nteger
Taklngthe productover . . ,|, . glvesz
/
=/or. F =
/ lt lollows that .dlvldes/,
ltnow suces toprove that /,_ .
Note that , ~: , = :
ltlollows that
,= ,+ : ,,
wherecr, l sapolynonlalwlth lntegercoeclents
ence
,+ t :

,,: :

= , ,:- i:

= ,: -, :+ l .
lorsonepolynonlal-,, wlthlnteger coeclents
Alsoobservethat, lorany xed
Thus
andwrltlng
.

= +.
-
: : - .
-
.

.
-
.
.
1
II6 4. 5o|ut|oostoAdvaoced Prob|e

we nd thatlor sone polynon|al /, lndependent o|. wlth loteger


coec|entssuch that
,:-.
-
, /,.
-
,

2
Putthen -: /, , - 2 |sdlv|slbleby ' -l and hencewecan wrlte
,-: , /,,2-,'-: ,,,
lorsonepolynonlal ,,w|th |nteger coeclents
Ralslngbothsldes to thepower q weobtaln
where, ,lsa polynonlal w|thlnteger coeclents
Lslng ,2) and (3) weobtaln
,+
,
,

-
, , , ,

-
, -: ,, ,
,-, ,'-: ,,,-,-: ,,
,,-,, ,-:, , , -,-: ,
-
,/,, - '

,
that |s,
'

= ;, , , -: ,

-
-,, ,- ,-

-
-: ,
where),,,and ,,,,arepolynonlalsw|th lnteger coec|ents
Eence/,_ '

, as deslred
0ur prool |sthus eonplete
Prohlcm 04 USAMO I008 proposal, Kiran Kcdlaya|
letbe aposlt|vereal nunber
a) Provethat
b) Prove that

- : , l
, - : , , -

C
, - : ,
`
:
-,-: , + ,

,+ /,

(3)
4. 5o|ut|oosto Advaoced Prob|ems
Solution 04
weuselnnlte telescop|ngsunsto solve theproblen
a) Lqu|valently, we haveto show that
Xote that
lt lollows that
C



:
,-: ,-

. .

, - , -
-
- -
,-
, - : , , - - : , , - : , , -
andthlstelescoplngsunnatlon ylelds thedcslred result
b) let
`
, - : ,
),,
-,-: , ,-,
:
Then. by a) . ), , < -.

II7
l npart|cular, ,,,converges to0 as approaches C sowe can
wrlte Jas an lnnltetelescoplng serles
`
), , _),-/- 1) - ),-/ ,
, ,
,
-,
0n the other hand, theresult |n a) glves
), :, ),,
,- : , :
- : ,
- - : ,
-
-
:
`
,- t ,

-t -

IIB 4. 5o|ut|oostoAdvaoced Prob|e

$ubstltutlngthe last equat-on to , I ) glves


as deslred
O
1
},:) =

,: +
_
)
:

Prohlcm 0 Bomania I000|


Letn lbeanlnteger,andlet
beasetol3n
:
elenents
Prove thatonecannd nlne dlstlnct nunbersc, , c, ,c, (i =l , ', l) lnX
such that thesysten
c; :-c,-c; :
c
:
x-c
:
-c
:
:
c,x-c, +c,:
has asolutlon :, ,v, ,z,) l nnonzerolntegers
Solution 0
o
o
o
Label the elenents olX lnlncreaslngorderx, < ' , , < x,- , andput
X; = |x; , . x-
],
X, = |x-+; , , x
:- ,
X = |x
:- +;

. x,- ]
lene thelunctlon } .X; ? X
:
7 X, " 5 ? 5 as lollows
}, c, c,c) = ,c c, c c)
The donalnol} contalnsnelenents
Therange ol}, on the other hand, ls contalned ln thesubset ol 5 ? 5
olpalrs whose sun ls atnost aset olcardlnallty
By thePigconholc Principlc, sone

hreetrlples ,c, , c, . c, ) ,i= I , ', l)


nap to thesane palr, .n whlchcase := c, c, ,v
=c,- c , : =c; - c,
lsasolutlon ln nonzerolntegers
_5o|ut|oos to Advaoced Prob|ems II9
Xote that c, cannot equal c
slnce X; and X
:
are dls]olnt, and that
al c
:
lnplles that the trlples ,c; ,c, ,cd and ,c
:
c
:
c
:
) are ldentlcal,
a
contradlctlon
Eence thenlnenunberschosen are .ndeed dlstlnct
Prohlcm 00 Xuanguo Huang|
Let n l bean lntegerand let x, ,x
:
,

, x- beposltlvereal nunbers
Suppose that
Prove that
-
I
-= I
I - :,
1 I I
-++_ ,n I )

- +
x; x, x-
Solution 00
Bysynnetry, wenayassune thatx; _ x
:
_

_ x- We have the
lollowlng lenna
Lcmma lor I _ i < _ n,
v v
-> -.
I+ x,

I +x,
PraaJ Slncen_ l, and, slnce
wehave
or
-
1
= I
1 -x,

|
I+ x,x, +x,x, > '+x,-x,
Itlollows thatx,x > I Thus
v _ v
=
I+x, 1 + x
=
v
, I+ x,
)-
v
.
+x,)
,I+x, ) , I -x )
,
v v
, I
v
, I -:, ) , I + :
:
)
_ 0,
I20
4. 5o|ut|oosto Advaoced Prob|e
ms
as deslred
Py thelenna,wehave
and,slnce
v
> > . . . >
v
I+x;
-
I+ x
:
- -
I+ x-

I I I
-> -> >
v v
-
v
|t lollowsby the Chchyshcv Incquality
Py heCauchy-Schwartz Incquality, wehave
or
Vultlplylngby
I
-
I
]
_

.,;
.
on both sldes ol 2) andapplylog I ) glves
whlehln turn lnpllesthedeslredlnequal l ty.
o
I )
2)
4. 5o|ut|oostoAdvaoced Prob|ems
Prohlcm 0?
Letx; ,x
:
, x- bed|stlnctreal nunbers.
Le|ne thepolynonlals
and
I,x) = ,: ::,: x
:
) ,: x-)
Q,x)=I, :)
I
+
_
+

+
I _

: x; x

x
:
x x-
Let v; v:
, , v-
-
; be therootso|Q
$how that
Solution 0?
nln

x
. x
:
< nln
v v)
,
= .=
Py synnetry, wenay assunethat
c=nln v, v
: =v:
v

,
=:
Let s =v; x , lor /=I 2, . ,
I2I
Pysynnetry, wenay also assune that s;
< s
:
< . . . < s- , l e , x; >
q_ > . . . > x-
lorthesakeolcootradlctloo, assune that
$lnce I has oo doubleroots, .tshares oone wlthQ
Then
or
I,v. )
+
+

+ =
'
.
=
'
I I I
v. x; v. -: v. x-
I I I
+ + . + ='
v. x; v. x
: v.
x-
ln partlcular, settlng |= I and i =2 glves
-
.
-
I
_
s
=
_
s+c
=
'

=; =;
I )
2)
We elaln that there|sa /such that s ,s+c)< 0, otherwlse,we have
I I
<
s +c s
1
22
4. 5o|ut|oos to Advaoccd Prob|cms
lor

all k, whlch ln turn lnpllesthat


whlch contradlcts 2)
-
l
-
l
_
s+ c

_
s

-; =:
Letbethenunbcrolkssuchthats,; +c) a thatl s, s a< +d
Aslnplebutcrltlcallactlsthats-cand s, have thesane slgn ln
lact, suppose that
bj s, s,+; s,+ as,++ , s-.
thcn
s, + c s, + cas,+j -c s- +c
Thcn s, > allandonly llk >i -I , thatlss +c>a
Fon ( 1 ) , wcobtaln s+ c _ s, , and,slnccs-cand s+ havc the
saneslgn,weobtaln
1 I

>
-
s+c

s.

lorall k= I , 2, ,

. Thcrclorc,
or
Also note that
-

:
1
-
-
l
_ = _
=;
s+
=;
s-c
,
1
-
l
_
- a
_

=;
s
=-
-
,+;
s+c

Addlng ,l)and !) ylclds


whlchcontradlcts 2)
-
1
-
i

s +c
=; =;
Thusourassunptlcn ls wrong and our proolls conp|cte
l)
!)
4. 5o|ut|oostoAdvaoccd Prob|cms 23
Prohlcm 08 Bomania I008|
$how thatorany posltlvc lnteger thepolynonla|
J,:)= :
:
--) +l
cannot bcwrlttcnas theproductoltwo non-constantpolynon|alswlth
lntegcrcoeclents
Solution 08
Xotethat},:) =p, /, :) ) , where/:) = :
:
+:andp,) =,

-l
$lncc
and
,
`, ls even lor l _ k _ .

- l p ls .rreduclble, by Llscnstcln' s
erlterlon
Xowletp beanon-constantlactor ol}, andletr bearootolp.
Thenp,/,r) ) = }r) =a so s =/r) ls arootolp
5lnces=r
-r E Qr) ,wehave Q,) C Q r) , so
degp degQr) ]Q) degQ) ]Q)= degp=2
-

Thusevery actor ol } hasdegreeat least 2


-

Therelore, ll} lsrcduelble,wecan wrlte },:)= A:)B,:) whcre^ and


B have dcgree 2
-

Xcxt, observe that


},:) & ,:
:
--
-l

-
''

+I
@
,:

+-+l ,

nod 2)
$lnce :
:
+:+1 lslrreduclbleln7

-| ,byunlquclactorlzatlon wc nust
have
Thus, ll wc wrlte
A,:)
B:)
~ c
:
:
:

+
+ a,.
|

+ - |, ,
then a

- , a

.. ,a,, |

- , |

.-. , |, areo ddand all the other cocc|ents


are cvcn 5lnce } ls nonlc, wc nay assune wlthout loss olgencrallty
124 4. 5o|ut|oosto Advaoced Prob|ems
that c,
= c,
= I also, c,c, = J0)= l . but c, > 0, c, > 0 as J has
noreal roots, so c, = c, = I
Thereore,
asc,. . c,. . lseven
But
and
,
.
;
) lsoddbyLucas's thcorcm, so
acontradlctlon
EenceJ| s|rreduclble
Prohlem 00 Iran I008|
Let
J ], l - l beunctlonssuch that
|snonoton|clorallc; ,c, .c, c R
Provethatthereexlst

, e

,e

c l, notall zero, sueh that


orallx C R
Solution 00, Altcrnativc I
Weestabl|shthe ollowlng lenna
Lcmma: LetJ,9 . l - l beunct|onssuch that Jlsnonconstantand
aJc}
|snonoton|corall c,cc R Thenthereex|stscc P such that
9 cJ |saconstantunct|on
PrccJ Let ,| betwo real nunberssuchthatJ, ) = J|)
4. 5o|ut|oos to Advaoced Prob|ems
125
Let
},) },|)

=
J, )

J |)

Let h = 9 cjJorsonecj cR
Then /, |snonoton|c But
/; , ) /; ,|)= },) },|) cj , ),) J,|))=
' }, ) J,|) ) , cj )
$|nce J, ) - J,|) = 0 | sxed, the nonoton|clty o hI depends onlyon
the slgnoU c
$|nceJ |s nonconstant, there ex|st x . x, c l such that J,xd = J,x, ) .
Let
andh = 9 cJ
Thenr= h( xt ) = /,x, ) andthenonotonlc|tyolh = 9 " c, ), oreach
c; , dependsonly on thes|gn olc

cj
We cla|nthath = 9 - cJ |sa constant unctl on
We prove our cla|n bycontrad|ctlon
$uppose,onthecontrary, thatthereexlstsx, E l suchthat/,x,) = r.
$|nceJ, x ) = J,x, ) , at least oneoJ,x ) = J,x,)andJ, x,) = J, x,) |s
true
Wlthout lossogeneral|ty,supposethatJ,x ) = J,x, )
Let

p, :
) p,x, )

=
J,x; )

J,x, )

Then thenonotonlcltyo hI also dependsonly on the s|gn oL


/
cj
$|nce/,x, ) = r= h(xt ) ,
hencec cj / L
/
- cj

$o there exsts sonecj such that / |sbothstr|ctly |ncreaslng and de-


creaslng,whlch |s |nposs|ble
Thereoreourassunpt|on|s lalseandh lsaconstantunct|on
Xowweproveournalnresult
I} ,], ] areallconstantunct|ons, theresult|s tr|v|al
Wlthoutlossogenerall ty, supposethat} |snonconstant
o
I2 4. 5o|ut|oostoAdvaoced Ptob|ems
lorc, ~ 0, we apply thelennato] and, so =c]+clorc, 0,
weapplythelennato] andJ

, so~ c' + c

Eerec, c . c.c areconstant


Wehave
,c c- c ) ]+ c c = ,c c- c ) ]+c ,c]+ c)- c,c]+c' ) o.
Il c'c- cc
,c
,-c); 0, 0, 0) , then let
andwe are done
0therwlse, c d' = 0and, ] are constant nultlplesol]
Then theproblen|saga|ntr|v|al
Solution 00, Altcrnativc 2
lenethe vector
t, x) = , ],:) , ],:) , ,:) )
lor : cR
llthet,:) spanapropersubspace olP
,
, we ca nd avector ,cj ,c, , c, )
orthogonal to that subspace, and then c, ,x) +c,], :) +c, ,:) = 0
lorall: cR
$o suppose thet,:)spanallolP
,

Then ther exlst xj < x, < x, c l such that t,:, v, x, ) , v,x,) are
llnearly lndependent, and so thc 3 7 3 natrlx A wlth Ai] = }, ,:.) has
llnearlyl ndependentrows
But thenA lslnvertlble,and lts colunnsalsospanR

Thls neanswecan nd c; , c, , c such that


,
_
c

,
!
, xj )
}. ,:, ) , }
.
,:, ) ) = 0, I , 0) ,
.=j
andtheunct|on ]+c,J,c

]lsthennotnonotonlc,acontradlctlon
4. 5o|ut|oos to Advaoced Ptob|ems I27
Prohlcm I00 USAMO I000 proposal, Bichard Stong|
Let x; , x, , , x- be varlables, and let vj , v, , ,v,=j be the sunsol
nonenptysubsetsox,
Let ,xj , .. :-) be the k' clcmcntary symmctric polynomial ln
thevthesunoevery producto|k dlstlnctv.

s)
lorwhlchkandnl severycoemclento, asapolynonlall nxj , ,x-)
even
Porexanple, ln ~ 2, then vj , v, , v, arexj ,x, ,xj +x, and
j = vj +v,+v, 2x; +2x, ,
, = v: v, +v,v,+v,v,
=:,+ : + 3
X1X, ,
,
=
v;v,v, x,x,+x; x
Solution I00
We say apolynonlal lsetenlevcry coemclentol l seven
0therwlse, wesay ls no| eten
lor any xed posltlve lnteger n, wesay a nonnegatlve lnteger k ls 'ac
Jorn lk =2
-
- 2

or sonenonnegatlvel nteger .
We wlllshowby lnductlononn that,xj ,x, ,
,:- ) ls notevenland
onlylk |s badlorn.
lor n ~ 1 , : = xj ls not even and k l ls bad or n = I as
k~ 1 ~ 2
;
2 =2
-
2
$upposethattheclalnlstrueoracertalnn.
We now conslder ,xj ,x, ,
,x-=j )
Let
, v; , v, , ,vs ) bethek' elenentarysynnetrlcpolynonlal
We havetheollowlnguseul , buteasy toprove, lacts
I
,v; ,v,, ,vs
,0) = , vj , v, ,
,v
)

2. lor allI
.. . = _ ,., v,

v-) , , v-+, ,

vs
)
i
.=)=
= ,x+v.,),:+v.2 )

,:+v..)

_ _ _
il <i2 < .. <ik =s SI <S2 <" ' <Sr
{ Sl , ' " , Sr} S{ .. .
,~-
YSI YS2

v
-
:
2B
Hence
4. 5o|ut|oos to Advaoced Prob|ems
-
. r
,
-~-
=
/ - r
- 4i 4s
-=a
(- ,i , +i )
_ ,( ,r . ,

:+,=-
, ,

i . i + +i i + ; +


+
+
-
) }
= _
_
2
_j (
,i )P- ,i )z,

,=- -=a J
By the induction hypothesis, every term of (- ,i , ,
) is even un
less r = 2

- 2' for some 0 _ t _ !.
For such r, note that
_
2

2'
j
- r
=
- r
is even unless - r = 0 or - r =2'
Therefore, taking coefcients modulo 2,
(- ,i ; . -+i )
_ ( ,i ; , , )( ,i ; . -)
.+,=-
By the induction hypothesis, the terms in the frst sum are even unless
k- 2

= 2

- 2" for some 0 : u _ ! that is / = 2

' - 2
In the second sum, every term appears twice except the term
for / even.
By the induction hypothesis, this term is even unless /J2 = 2

- 2 . for
some 0 _ L _ n, that is / = 2
+i
2

i

It follows that
(-
i ,,


+i ) is even unless / =2
+
-
- 2" for some
0 : O _ ! + I , i. e. , / is bad for n + 1 :
4. 5o|ut|oos to Advaoced Prob|ems
29
Furthermore, note that the odd coefcients in
occur for diferent powers of + r
Therefore, the condition that k is bad for + 1 is also sufcient for
to be odd.
Our induction is complete.
Prohlcm I01 Bussia 2000|
Prove that there exist 10 di stinct real numbers a
i , a, ,
, ai such that
the equation
has exactly 5 diferent real roots.
Solution I0I
We show that [ai a,

a
i: = [ I 0, . -2} i s a group of numbers
satisfying the conditions given in the problem.
The given equality becomes
,z ~ 2) ,- - l) x(x + l) (x - 2

.
,
) 0,
where
,u) = 2 ,,I - 0 + +
`
) u` +
,I 0 5 + I 0 4 + . + 4
`
u + 0
`


If ,u) =has no real solutions, then ,
;
) = 0 has no real solutions.
If u, and u; are real solutions of , u) =0, then ui u, < 0 and ui u; > 0,
that is, both ui and u; are negative.
It follows again that ,
;
) 0 has no real solutions.
Our proof is complete.
L55A!Y
Arithmctic-Gcomctric Mcan Incquality [AMGM Incquality)
Iaj , a, , , a- aren nonnegatlve nunbers, then
wlth equal|ty landonly |al = a, = ~ a-

DinomiaI Cocmcicnt
The coeclentoX
k
|n the expanslono ,: I )
-
ls
n n'
k

k' n ~ k) '
Cauchy-Schwarz Incquality
loranyrealnunbers a ,a, ,. . ,a-, andcj , c, , ,c-
w|thequal|tylandonly |a, and c, areproport|onal, i ~ 1, 2, , n
Chchyshcv Incquality
I . Let Xl , x

. , x, and Yl , v, , . . , v- be two sequencesoreal nun-


bers, such that Xl 5 X2 5

; x, and vl 5 v, 5 . 5 v-
Then
2. Let Xl , x

. , x, and vl ,v, , , v- betwo sequencesoreal nun-


bers, such that Xl x, x, and
vl v,
v-
Then
I32 C|ossary
Dc Moivrc's Formula
loranyanglc L and loranyl ntcgcr n
,
cos o+i slno)
-
= cosno+ i slnno
Elcmcntary Symmctric Polynomials (Functions)
Clvcn lndctcrnlnatcs :, , , :- , thc clcncntary synnctrlc lunct|ons
s , ,

,s-
arcdcncdbythcrclatlon |n anothcrlndctcrn|natc|)
|x, )

,|+ :-) = |
-
-s, |
,
-
;
+ . + --.; | +--
That ls. s, ls thc sun olthc products ol thc :, takcn k at a tlnc lt
|s a baslc rcsult that cvcry synnctrlc polynon|al ln x, ,
,:- can bc
unlqucly) cxprcsscdas apolynonlal ln thc s, , and vlcc vcrsa
Fibonacci Numbcrs
$cqucnccdchncd rccurslvclybyF; = F, = 1, F-,, = F-=; + F- , lorall
nc N.
3cnscn's IncquaIity
Il}lsconcavcuponanlntcrva| c,c[ and; , , ,

, -arcnonncgatlvc
nunbcrswlth sun cqual to I , thcn
, :; ) ,] ,:, )

-],:-) }', x; + ,:,+ . . . + -:-)


lor any :, , :,, ,:- |n thc ntcrval c.|[ Il thc lunctlon ls concavc
down, thc|ncqual|ty |srcvcrscd
Lagrangc's Intcrpolation Formula
L

t :, , :; , ,:- bcdlstlnct rcal nunbcrs, and lct v, . vj , ,v- bcar-


b|trary rcal nunbcrs Thcn thcrc cxlsts a unlquc polynonlal I, :) ol
dcgrcc at nost n such that I,:, ) = v.
, i = 0. I , ,n Thls ls thc
polynonlal glvcnby
j
, :) =
_
Y
i
, : :,)

,: :,
-
j) |:- :, =; )


,: :-)

,s
,:, :, ) ':, ~ :, ; ) :, :,=

,:, :-)
Law of Cosincs
.ct AEC bca trlanglc Thcn
EC
,
AE
,
AC

- 2AE ACcos A
C|ossary I33
Lucas' Thcorcm
Lct bcapr|nc lctc and c bctwoposltlvclntcgcrs such that
c= c` c. ;

-
'

c;c,,c= c

c;

-
;

c;+ c, ,
whcrc0_ c, , c, < arc lntcgcrslor i= 0,I , . k Thcn

c
;~
c

;
c
-
;
;
c

;
c,
; nod)

c c c
-
j c; c,
Pigconholc Principlc
lln ob]cctsarc dlstrlbutcd anong k < n boxcs, sonc box contalns at
|casttwo ob]ccts
Boot Mcan Squarc-Arithmctic Mcan Incquality (BMS-AM In-
cquaIity)
lorposltlvcnunbcrs:, ,:, , :-,

::

. + :

:, + :, + . . .
+ :

n n
Vorc gcncrally, lctc; ,c, , ,c- bcany posl tlvcnunbcrs|orwhlchc;
c,-

c- = 1. lor posl tlvc nunbcrs:j ,:, ,

:- wc dchnc
M
-
_ = uin|:; , :, , , : ,
M_ =naxr, ,: , ,:
M, =: :

: ,
M, = ,c; :+ c:+ c:,)
j
' ,
whcrct lsanon-zcrorca|nunbcr Thcn
lors _ t .
I34 C|ossary
Tianglc Incquality
Let Z = a|i be a conplex nunber. lehne the absolutevalueoz to
be
I zl
= va
2
+6
2
.
Let a and ( betwoconplexnunbers Thelnequallty
,(
1

l
al +

ls called the tr|angle|nequal|ty


Leta = al +a
2
i and( = (1 (
2
i , whereaI , a
2
, (1 , (
2
arerealnunbers
Thena 0= (al +(1 ) (
a
2 +(2 )
i .
Vectors u = o , o_| , ^ = , 0_| , and = a , o_ (2] orn a
trlangle wlth s|deslengths,,,
0 , and,+(1 .
Thetrlanglelnequallty restates theact that thelength oany s|deoa
tr|angle|sless than the sun othelengthsothe other two sldes
Victa's Thcorcm
LetXl , X2
, . . . , Xn betherootsopolynonlal
Px)= anx
n
+an_l X
n
-
l

.
a; r+a,
where a
n
= 0and a, ,a, , ,c- cl. Let Sk
bethe sun otheproducts
otheXi taken/ ata t|ne. Then
thatl s,
l )

a
n-k
Sk
= -

a
n
an-l
Xl +X2 +

Xn = -;
an
an-2
Xl X2 +

+XiX
j
Xn
-
lXn = ;
an
Tigonomctric Idcntitics
s|n

a+cos

a= 1,
sln a
tanX = , ,
cosa
I
cct x=
tana
'
C|ossary
add|t|onand subtract|on ornula
s|na+|) = s|nacos|+cos a sln|,
cosa+|) = cosacos|= s|nasln|,
tana+tan |
tana + |) =
I t t |

= an a an
double-angleornulas
s|n 2a= 2 sln a cos a,
cos 2a= cos

a~ s|n

a= 2 cos

a 1 = 1 - 2 s|n

a,
2 tan a
tan2a=
1
2
'
- tan a
trlple-angleornula.
hal|-angleornula
s|n3a= 3s|na~ 1 sln

a,
cos3a= 1cos

a- 3cosa,
3 tan a- tan

a
tan3a=
I 3tan
2
a

2tan
sna=
1
2 a '
+ tan '
1 - tan

cosa=
1 t
2 a '
+ an '
2 tan
tana=
I t
2 a ;
- an '
sum-toproductornula.
a + | a - |
s|n a+s|n |= 2s|n*
2
-cos*
2

a | a - |
cos a+cos|= 2 cos*
2
cos*
2

sina|)
tana+tan|=
|

cos a cos
d|herence-to-product|ornula
a - | a + |
slna~ sln|= 2sln*
2
-cos*
2

I35
I3
b 2

a - b . a + b
cos a- cos - sa
2
su
2

sln(a - b)
tana - tanb ;
cosa cosb
product-to-sun lornulas.
2sln acos |= sln a+ b) + slna- b) ,
2cosacosb cos(a + b) + cos(a - b) ,
2slnU slnb - cosa + b) + cosa - b) .
C|ossary

IUHHLH HLAJN
I. Andrccscu,T Kcdlaya, K; 2cltz, P , Mathematical Contests
1995-1 996: Olympiad Problems from around the World, with
Solutions, Ancrlcan \lathcnatlcsConpctltlons, I
2. Andrccscu, T Kcdlaya. K, Mathematical Contests 1996- 1997:
Olympiad Problems fom around the World, with Solutions,
Ancrlcan VathcnatlcsConpctltlons, I3
3. Andrccscu, T Kcdlaya. K, Mathematical Contests 1997-1 998:
Olympiad Problems from around the World, with Solutions,
Ancrlcan \lathcnatlcs Conpctl tlons, I
4. Andrccscu, T lcng, 2 . Mathematical Olympiads: Problems and
Solutions from around the World, 1998-1999, \lathcnatl cal
Assoclatlon o!Ancrlca, 2000
. Andrccscu, T Cclca, R , Mathematical Olympiad Callenges,
Blrkhauscr, 2000
0. Barbcau, L , Polynomials , $prlngcr-Vcr|ag, I3
1. Bcckcnbach, L l , Bcllnan, R . An Introduction to Inequalities,
XcwVathcnatlcal L|brary, Vol 3, VathcnatlcalAssoclatlonol
Ancrlca, II
8 Chnn, W. C , $tecnrod, X L , First Concepts of Topology, Xcw
VathcnatlcalL|brary, Vo| 2, RandonEousc, l
0. Colnan, J. , Wat to Solve ?, 0xlord $clcnccPubllcatlons, I 0
I0 Coxctcr. E $. V , Orcltzcr, $. L , Geometry Revisited, Xcw
VathcnatlcalL| brary, Vol I, Vathcnatlca| Assoclatlon ol
Ancrlca, I
II. Doob, V , The Canadian Mathematical Olympiad 1969-1 993,
LnlvcrsltyolToronto Prcss, I3
I2. Lngcl, A , Problem-Solving Strategies, ProblcnBooksln
Vathcnatlcs, $prlngcr, l3
I3. lonln, D. , Klrlchcnko. A , Leningrad Mathematical Olympiads
1987-1 991 , VathPro Prcss. I!
3B Further Pead|n
I4. Fomin, D. , Genkin, S. , Itenberg, I. , Mathematical Circles ,
American Mathematical Soci ety, I
I. Graham, R. L. , Knuth, D. E. , Patashnik, O , Concrete
Mathematics , Addison-Wesley, l 3
I0. Greitzer , S. L. , Interational Mathematical Olympiads, 1959-1 977,
New Mathematical Li brary, Vol . 2, Mathematical Association of
America, I3
17. Grossman, I . , Magnus, W. , Groups and Their Graphs, New
Mathematical Library, Vol. 14, Mathematical Association of
America, I1
I8. Kazarinof, N. D. , Geometric Inequalities, New Mathematical
Li brary, Vol. 1, Random House, I I
)0. Klamkin, M. , Interational Mathematical Olympiads, 1978- 1985 ,
New Mathematical Library, Vol. 3I , Mathematical Association of
America, I3
20. Klamkin, M. , USA Mathematical Olympiads, 1972-1986, New
Mathematical Library, Vol . 33. Mathematical Association of
America, I 33
2I. Kirschak, J. , Hungarian Problem Book, Volumes I C I, New
Mathematical Library, Vols. I I I 2, Mathematical Association of
America, I
22. Kuczma, M. , 144 Problems of the Austrian-Polish Mathematics
Competition 1978-1993, The Academic Distribution Center, I 1
23. Larson, L. C. , Problem-Solving Through Problems, Springer-Verlag,
I33.
24. Lausch, H. , Bosch Giral, C. , The Asian Pacifc Mathematics Olym
piad 1989-2000, AMT Publishing, Canberra, 200I
26. Liu, A. , Chinese Mathematics Competitions and Olympiads
1981- 1993, AMT Publishing, Canberra, I3
20. Lozansky, E. , Rousseau, C. Winning Solutions, Springer, I
27. Ore, O. , Grphs and their uses, Random House, I3

28. Ore, O. , Invitation to number theory, Random House, I


20. Sharygin, I. l , Problems in Plane Geometry, Mir, Moscow, I33

Further Pead|n
139
30. Sharygin, I. F. , Problems in Solid Geometry, Mir, Moscow, I 3
3I. Shklarsky, D. O, Chentzov, N. N; Yaglom, I. M. , The USSR
Olympiad Problem Book, Feeman, I2
32. Slinko, A. , USSR Mathematical Olympiads 1989- 1992, AMT
Publishing, Canberra, I
33. Soifer, A. , Colorado Mathematical Olympiad: The frst ten years,
Center for excellence in mathematics education, I 1
34. Szekely, G. J. , Contests in Higher Mathematics, Springer- Verlag,
I
3. Stanley, R. P. , Enumerative Combinatorics, Cambridge University
Press, I
30. Taylor, P. J. , Tourament of Towns 1980-1 984 , AMT Publishing,
Canberra, I 3
3?. Taylor, P. J. , Tourament of Towns 1984-1 989, AMT Publishing,
Canberra, I 2
38. Taylor, P. J. , Tourament of Towns 1989-1 993, AMT Publishing,
Canberra, I 1
39. Taylor, P. J. , Storozhev, A. , Tourament of Towns 1993-1997,
AMT Publishing, Canberra, I 3
40. Tomescu, I. , Problems in Combinatorics and Graph Theory, Wiley
I3
4I. Vanden Eynden, C. , Elementary Number Theory, McGraw-Hill,
I3
42. Wilf, H. S. , Generatingfunctionology, Academic Press, I1
43. Wilson, R. , Introduction to graph theory, Academic Press, I2
44. Yaglom , 1. M. , Geometric Tansformations, New Mathematical
Library, Vol . 3, Random House, I2
4. Yaglom , I. M. , Geometric Tansformations I, New Mathematic
Library, Vol . 2I , Random House, I 3
40. Yaglom , I. M. , Geometric Tansformations II, New Mathematir
Library, Vol. 21, Random House, I3
4?. Zeitz, P. , The Art and Craf of Problem Solving, John Wiley
Sons, I
THE AUS TRAL I AN MATH E MATI CS TRUST
E N R l f H M E N T S E R I E S
Titu Andreescu is
Director of the
A m e r i c a n
M a t h e m a t i c s
Co m p e i t i Un 5 y
seres as Head
Coach of the USA
I n t e r n a t i o n a l
Ma t h e ma t i c a l
Olympiad (jMO)
Team, is
Chair of the US Mathematical
Olympiad Committee, and is Director of the
USA Mathematical Olympiad Summer
Program. Originally from Romania, Prof.
Andreescu received the Distinguished
Professor Award from the Romanian
MinistT of Education in 1 983, then after
mong to the USA was awarded the Edyth
May Slife Award for Distinguished High
School Mathematics Teaching from the
MM in 1 994. In addition, he received a
Certifcate of Appredation presented by the
President of the MM fOT Mhis outstanding
src as coach of the USA Mathematical
Olympiad Progrm in preparng the USA
team for its perfect perforance in Hong
Kong at the 1 994 lMO.
Zuming Feng
graduated with a
PhD from Johns
Hopkins University
with an" emphasis
on Algebraic
Number Theor
and
!
Elliptic
CUN. He teaches
at Phillips Exeter
Academy. He also seres as 4 coach of the
USA Interational Mathematical Olympiad
(IMO) Team, is a member' of the USA
Mathematical Olympiad Committee, and is
an assistant director of the USA
Mathematical Olympiad Summer Program.
He received the Edyth May Slife Award for
Distinguished High School Mathematics
Teaching from the MM in 1 996.
I SBN 8 7 6 42 0 1 2 X

S-ar putea să vă placă și